Зная площадь как узнать стороны прямоугольника: «Как зная площадь прямоугольника найти стороны?» – Яндекс.Кью

Содержание

Найти стороны прямоугольника известна только площадь. Периметр и площадь прямоугольника

Площадь прямоугольника, как не будет дерзко звучать, но это важное понятие. В повседневной жизни мы постоянно сталкиваемся с ним. Узнать размер полей, огородов, рассчитать количество краски, необходимой для побелки потолка, сколько понадобится обоев для оклейки ко

мнаты и другое.

Геометрическая фигура

Для начала поговорим о прямоугольнике. Это фигура на плоскости, которая имеет четыре прямых угла, а ее противоположные стороны равны. Стороны его привыкли называть длиной и шириной. Измеряют их в миллиметрах, сантиметрах, дециметрах, метрах и т. д. Теперь ответим на вопрос: «Как найти площадь прямоугольника?» Для этого необходимо длину умножить на ширину.

Площадь=длина*ширина

Но еще одна оговорка: длина и ширина должны быть выражены в одинаковых единицах измерения, то есть метр и метр, а не метр и сантиметр. Записывается площадь латинской буквой S. Для удобства обозначим длину латинской буквой b, а ширину латинской буквой a, как показано на рисунке. Отсюда мы делаем вывод, что единицей измерения площади является мм 2 , см 2 , м 2 и т. д.

Рассмотрим на конкретном примере, как найти площадь прямоугольника. Длина b=10 ед. Ширина a=6 ед. Решение: S=a*b, S=10 ед.*6 ед., S=60 ед 2 . Задача. Как узнать площадь прямоугольника, если длина в 2 раза больше ширины и составляет 18 м? Решение: если b=18 м, тогда а=b/2, a=9 м. Как найти площадь прямоугольника, если известны обе стороны? Правильно, подставить в формулу. S=a*b, S=18*9, S=162 м 2 . Ответ: 162 м 2 . Задача. Сколько необходимо купить рулонов обоев для комнаты, если ее размеры составляют: длина 5,5 м ширина 3,5, а высота 3 м? Размеры рулона обоев: длина 10 м, ширина 50 см. Решение: сделаем рисунок комнаты.

Площади противоположных сторон равны. Вычислим площадь стены с размерами 5,5 м и 3 м. S стены 1 =5,5*3,

S стены 1 =16,5 м 2 . Следовательно, противоположная стена имеет площадь равную 16,5 м 2 . Найдем площади следующих двух стен. Стороны их, соответственно, равны 3,5 м и 3 м. S стены 2 =3,5*3, S стены 2 =10,5 м 2 . Значит, и противоположная сторона равна 10,5 м 2 . Сложим все результаты. 16,5+16,5+10,5+10,5=54 м 2 . Как вычислить площадь прямоугольника, если стороны выражены в разных единицах измерения. Ранее мы вычисляли площади в м 2 , то и в этом случае будем использовать метры. Тогда ширина рулона обоев будет равна 0,5 м. S рулона =10*0,5, S рулона =5 м 2 . Теперь узнаем, сколько рулонов необходимо для оклейки комнаты. 54:5=10,8 (рулонов). Так как они измеряются целыми числами, то нужно купить 11 рулонов обоев. Ответ: 11 рулонов обоев. Задача. Как вычислить площадь прямоугольника, если известно, что ширина на 3 см короче длины, а сумма сторон прямоугольника составляет 14 см? Решение: пусть длина х см, тогда ширина (х-3) см. х+(х-3)+х+(х-3)=14, 4х-6=14, 4х=20, х=5 см — длина прямоугольника, 5-3=2 см — ширина прямоугольника, S=5*2, S=10 см 2 Ответ: 10 см 2 .

Резюме

Рассмотрев примеры, надеюсь, стало понятно, как найти площадь прямоугольника. Напомню, что единицы измерения длины и ширины должны совпадать, иначе получится неправильный результат, чтобы не допустить ошибок, читайте задание внимательно. Иногда сторона может быть выражена через другую сторону, не стоит бояться. Обратитесь к нашим решенным задачам, вполне возможно, они могут помочь. Но хоть раз в жизни мы сталкиваемся с нахождением площади прямоугольника.

4. Формула радиуса окружности, которая описана около прямоугольника через диагональ квадрата :

5. Формула радиуса окружности, которая описана около прямоугольника через диаметр окружности (описанной):

6. Формула радиуса окружности, которая описана около прямоугольника через синус угла, который прилегает к диагонали, и длину стороны противолежащей этому углу:

7. Формула радиуса окружности, которая описана около прямоугольника через косинус угла, который прилегает к диагонали, и длину стороны у этого угла:

8. Формула радиуса окружности, которая описана около прямоугольника через синус острого угла между диагоналями и площадью прямоугольника:

Угол между стороной и диагональю прямоугольника.

Формулы для определения угла между стороной и диагональю прямоугольника:

1. Формула определения угла между стороной и диагональю прямоугольника через диагональ и сторону:

2. Формула определения угла между стороной и диагональю прямоугольника через угол между диагоналями:

Угол между диагоналями прямоугольника.

Формулы для определения угла меж диагоналей прямоугольника:

1. Формула определения угла меж диагоналей прямоугольника через угол между стороной и диагональю:

β = 2α

2. Формула определения угла между диагоналями прямоугольника через площадь и диагональ.

С таким понятием, как площадь, нам приходится сталкиваться в своей жизни повседневно. Так, например, при строительстве дома ее нужно знать для того, чтобы рассчитать количество необходимого материала. Размер садового участка также будет характеризоваться площадью. Даже ремонт в квартире невозможно сделать без этого определения. Поэтому вопрос, как найти площадь прямоугольника, на нашем встает очень часто и является важным не только для школьников.

Для тех, кто не знает, прямоугольник — это плоская фигура, у которой противоположные стороны равны, а углы составляют 90о. Для обозначения площади в математике используют английскую букву S. Ее измеряют в квадратных единицах: метрах, сантиметрах и так далее.

Теперь попытаемся дать подробный ответ на вопрос, как найти площадь прямоугольника. Существует несколько способов определения этой величины. Наиболее часто мы сталкиваемся со способом определения площади с помощью ширины и длины.

Возьмем прямоугольник с шириной b и длиной k. Для вычисления площади данного прямоугольника необходимо ширину умножить на длину. Это все можно представить в виде формулы, которая будет выглядеть так: S = b * k.

А теперь рассмотрим этот способ на конкретном примере. Необходимо определить площадь садового участка с шириной 2 метра и длиной 7 метров.

S = 2 * 7 = 14 м2

В математике, особенно в приходится определять площадь иными способами, так как во многих случаях ни длина, ни ширина прямоугольника нам не известна. Вместе с тем имеют место другие известные величины. Как найти площадь прямоугольника в этом случае?

  • Если нам известна длина диагонали и один из углов, составляющий диагональ с любой стороной прямоугольника, то в этом случае потребуется вспомнить о площади Ведь если разобраться, то прямоугольник состоит из двух равных прямоугольных треугольников. Итак, вернемся к определяемой величине. Для начала необходимо определить косинус угла. Полученную величину умножить на длину диагонали. В итоге получим длину одной из сторон прямоугольника. Аналогично, но уже с помощью определения синуса, можно определить длину второй стороны. А как найти площадь прямоугольника теперь? Да очень просто, перемножить полученные величины.

В виде формулы это будет выглядеть так:

S = cos(a) * sin(a) * d2 , где d- длина диагонали

  • Еще один способ определения площади прямоугольника — через вписанную в него окружность. Он применяется в том случае, если прямоугольник является квадратом. Для использования данного способа необходимо знать Как вычислить площадь прямоугольника таким способом? Конечно же, по формуле. Доказывать мы ее не будем. А выглядит она так: S = 4 * r2, где r -радиус.

Случается так, что вместо радиуса нам известен диаметр вписанной окружности. Тогда формула будет выглядеть так:

S=d2,где d — диаметр.

  • Если известна одна из сторон и периметр, то как узнать площадь прямоугольника в этом случае? Для этого необходимо произвести ряд простых вычислений. Как мы знаем, противоположные стороны прямоугольника равны, поэтому от значения периметра необходимо отнять известную длину, умноженную на два. Полученный результат разделить на два и получим длину второй стороны. Ну, а дальше стандартный прием, перемножаем обе стороны и получаем площадь прямоугольника. В виде формулы это будет выглядеть так:

S=b* (P — 2*b), где b — длина стороны, P — периметр.

Как видим площадь прямоугольника можно определять различными способами. Все зависит от того, какие величины нам известны перед рассмотрением данного вопроса. Конечно же, последние методы исчисления в жизни практически не встречаются, но могут пригодиться для решений многих задач в школе. Возможно, и для решения ваших задач эта статья окажется полезной.

Определение.

Прямоугольник
— это четырехугольник у которого две противоположные стороны равны и все четыре угла одинаковы.

Прямоугольники отличаются между собой только отношением длинной стороны к короткой, но все четыре угла у них прямые, то есть по 90 градусов.

Длинную сторону прямоугольника называют длиной прямоугольника
, а короткую — шириной прямоугольника
.

Стороны прямоугольника одновременно является его высотами.

Основные свойства прямоугольника

Прямоугольником могут быть параллелограмм, квадрат или ромб.

1. Противоположные стороны прямоугольника имеют одинаковую длину, то есть они равны:

AB = CD, BC = AD

2. Противоположные стороны прямоугольника параллельны:

3. Прилегающие стороны прямоугольника всегда перпендикулярны:

AB ┴ BC, BC ┴ CD, CD ┴ AD, AD ┴ AB

4. Все четыре угла прямоугольника прямые:

∠ABC = ∠BCD = ∠CDA = ∠DAB = 90°

5. Сумма углов прямоугольника равна 360 градусов:

∠ABC + ∠BCD + ∠CDA + ∠DAB = 360°

6. Диагонали прямоугольника имеют одинаковой длины:

7. Сумма квадратов диагонали прямоугольника равны сумме квадратов сторон:

2d
2 = 2a
2 + 2b
2

8. Каждая диагональ прямоугольника делит прямоугольник на две одинаковые фигуры, а именно на прямоугольные треугольники.

9. Диагонали прямоугольника пересекаются и в точке пересечения делятся пополам:

10. Точка пересечения диагоналей называется центром прямоугольника и также является центром описанной окружности

11. Диагональ прямоугольника является диаметром описанной окружности

12. Вокруг прямоугольника всегда можно описать окружность, так как сумма противоположных углов равна 180 градусов:

∠ABC = ∠CDA = 180° ∠BCD = ∠DAB = 180°

13. В прямоугольник, у которого длина не равна ширине, нельзя вписать окружность, так как суммы противоположных сторон не равны между собой (вписать окружность можно только в частный случай прямоугольника — квадрат).

Стороны прямоугольника

Определение.

Длиной прямоугольника
называют длину более длинной пары его сторон. Шириной прямоугольника
называют длину более короткой пары его сторон.

Формулы определения длин сторон прямоугольника

1. Формула стороны прямоугольника (длины и ширины прямоугольника) через диагональ и другую сторону:

a
= √d
2 — b
2

b
= √d
2 — a
2

2. Формула стороны прямоугольника (длины и ширины прямоугольника) через площадь и другую сторону:

Диагональ прямоугольника

Определение.

Диагональю прямоугольника
называется любой отрезок соединяющий две вершины противоположных углов прямоугольника.

Формулы определения длины диагонали прямоугольника

1. Формула диагонали прямоугольника через две стороны прямоугольника (через теорему Пифагора):

d
= √a
2 + b
2

2. Формула диагонали прямоугольника через площадь и любую сторону:

4. Формула диагонали прямоугольника через радиус описанной окружности:

d
= 2R

5. Формула диагонали прямоугольника через диаметр описанной окружности:

d
= D о

6. Формула диагонали прямоугольника через синус угла, прилегающего к диагонали, и длину стороны противоположной этому углу:

8. Формула диагонали прямоугольника через синус острого угла между диагоналями и площадью прямоугольника

d
= √2S: sin β

Периметр прямоугольника

Определение.

Периметром прямоугольника
называется сумма длин всех сторон прямоугольника.

Формулы определения длины периметру прямоугольника

1. Формула периметру прямоугольника через две стороны прямоугольника:

P = 2a
+ 2b

P = 2(a
+ b
)

2. Формула периметру прямоугольника через площадь и любую сторону:

P =2S + 2a
2
=2S + 2b
2
a

b

3. Формула периметру прямоугольника через диагональ и любую сторону:

P = 2(a
+ √d
2 — a
2
) = 2(b
+ √d
2 — b
2
)

4. Формула периметру прямоугольника через радиус описанной окружности и любую сторону:

P = 2(a
+ √4R 2 — a
2
) = 2(b
+ √4R 2 — b
2
)

5. Формула периметру прямоугольника через диаметр описанной окружности и любую сторону:

P = 2(a
+ √D o 2 — a
2
) = 2(b
+ √D o 2 — b
2
)

Площадь прямоугольника

Определение.

Площадью прямоугольника
называется пространство ограниченный сторонами прямоугольника, то есть в пределах периметра прямоугольника.

Формулы определения площади прямоугольника

1. Формула площади прямоугольника через две стороны:

S = a · b

2. Формула площади прямоугольника через периметр и любую сторону:

5. Формула площади прямоугольника через радиус описанной окружности и любую сторону:

S = a
√4R 2 — a
2
= b
√4R 2 — b
2

6. Формула площади прямоугольника через диаметр описанной окружности и любую сторону:

S = a
√D o 2 — a
2
= b
√D o 2 — b
2

Окружность описанная вокруг прямоугольника

Определение.

Окружностью описанной вокруг прямоугольника
называется круг проходящий через четыре вершины прямоугольника, центр которого лежит на пересечении диагоналей прямоугольника.

Формулы определения радиуса окружности описанной вокруг прямоугольника

1. Формула радиуса окружности описанной вокруг прямоугольника через две стороны:

4a, где a — сторона квадрата или ромба. Тогда длина стороны
равна одной четвертой периметра: a = p/4.

Легко разрешима эта задача и для треугольника. У него три одинаковых по длине стороны
, поэтому периметр p равностороннего треугольника равен 3a. Тогда сторона равностороннего треугольника a = p/3.

Для остальных фигур понадобятся дополнительные данные. Например, можно найти стороны
, зная его периметр и площадь. Предположим, что длина двух противолежащих сторон прямоугольника равна a, а длина двух других сторон — b. Тогда периметр p прямоугольника равен 2(a+b), а площадь s равна ab. Получим систему с двумя неизвестными:
p = 2(a+b)
s = ab.1/2)/2. Отбросьте тот корень, который будет меньше ноля, и подставьте в для стороны
a.

Источники:

  • Найти стороны прямоугольника

Если вы знаете значение а, то вы можете сказать, что решили квадратное уравнение, потому как его корни будут найдены очень легко.

Вам понадобится

  • -формула дискриминанта квадратного уравнения;
  • -знание таблицы умножения

Инструкция

Видео по теме

Полезный совет

Дискриминант квадртаного уравнения может быть положительным, отрицательным, или равняться 0.

Источники:

Частный случай параллелограмма — прямоугольник – известен только в геометрии Евклида. У прямоугольника
равны все углы, и каждый из них по отдельности составляет 90 градусов. Исходя из частных свойств прямоугольника
, а также из свойств параллелограмма о параллельности противолежащих сторон можно найти стороны
фигуры по заданным диагоналям и углу от их пересечения. Вычисление сторон прямоугольника
основывается на дополнительных построениях и применении свойств получаемых фигур.

Инструкция

Буквой А отметьте точку пересечения диагоналей. Рассмотрите образованный построениями EFА. Согласно свойству прямоугольника
его диагонали равны и пополам точкой пересечения А. Вычислите значения FА и EА. Так как треугольник EFА равнобедренным и его стороны
EА и FА равны между собой и соответственно равны половине диагонали EG.

Далее вычислите первую EF прямоугольника
. Данная сторона является третьей неизвестной стороной рассматриваемого треугольника EFА. Согласно теореме косинусов по соответствующей формуле найдите сторону EF. Для этого подставьте в формулу косинусов полученные ранее значения сторон FА EА и косинус известного угла между ними α. Вычислите и запишите полученное значение EF.

Найдите вторую сторону прямоугольника
FG. Для этого рассмотрите другой треугольник EFG. Он является прямоугольным, где известны гипотенуза EG и катет EF. Согласно теореме Пифагора найдите второй катет FG по соответствующей формуле.

Равносторонний треугольник наряду с квадратом является, пожалуй, самой простой и симметричной фигурой в планиметрии. Разумеется, все соотношения, справедливые для обычного треугольника, верны также и для равностороннего. Однако для правильного треугольника все формулы становятся намного проще.

Вам понадобится

  • калькулятор, линейка

Инструкция

Чтобы измерьте длину одной из его сторон и умножьте результат измерения на три. В виде это можно записать следующим образом:

Прт = Дс * 3,

Прт – периметр треугольника,
Дс – длина любой из его сторон.

Периметр треугольника получится в тех же измерения, что и длина его стороны.

Так как равносторонний треугольник обладает высокой степенью симметрии, то для вычисления его периметра достаточно одного из параметров. Например, площади, высоты, вписанной или описанной окружности.

Если известен радиус вписанной окружности равностороннего треугольника, то для вычисления его периметра воспользуйтесь следующей формулой:

Прт = 6 * √3 * r,

где: r — радиус вписанной окружности.¼.

Видео по теме

Квадрат представляет собой геометрическую фигуру, состоящую из четырех сторон одинаковой длины и четырех прямых углов, каждый из которых равен 90°. Определение площади или периметра

четырехугольника, причем любого, требуется не только при решении задач по геометрии, но и в повседневной жизни. Эти умения могут стать полезными, например, во время ремонта при расчете нужного количества материалов — покрытий для пола, стен или потолка, а также для разбивки газонов и грядок и т.д.

7 способов найти площадь прямоугольника

1. Если известны две соседние стороны

Просто перемножьте две стороны прямоугольника.

  • S — искомая площадь прямоугольника;
  • a и b — соседние стороны.

2. Если известны любая сторона и диагональ

Найдите квадраты диагонали и любой стороны прямоугольника.

От первого числа отнимите второе и найдите корень из результата.

Умножьте длину известной стороны на полученное число.

  • S — искомая площадь прямоугольника;
  • a — известная сторона;
  • d — любая диагональ (напомним: обе диагонали прямоугольника имеют одинаковую длину).

3. Если известны любая сторона и диаметр описанной окружности

Найдите квадраты диаметра и любой стороны прямоугольника.

От первого числа отнимите второе и найдите корень из результата.

Умножьте известную сторону на полученное число.

  • S — искомая площадь прямоугольника;
  • a — известная сторона;
  • D — диаметр описанной окружности.

4. Если известны любая сторона и радиус описанной окружности

Найдите квадрат радиуса и умножьте результат на 4.

Отнимите от полученного числа квадрат известной стороны.

Найдите корень из результата и умножьте на него длину известной стороны.

  • S — искомая площадь прямоугольника;
  • a — известная сторона;
  • R — радиус описанной окружности.

5. Если известны любая сторона и периметр

Умножьте периметр на длину известной стороны.

Найдите квадрат известной стороны и умножьте полученное число на 2.

От первого произведения отнимите второе и разделите результат на 2.

6. Если известны диагональ и угол между диагоналями

Найдите квадрат диагонали.

Разделите полученное число на 2.

Умножьте результат на синус угла между диагоналями.

  • S — искомая площадь прямоугольника;
  • d — любая диагональ прямоугольника;
  • α — любой угол между диагоналями прямоугольника.

7. Если известны радиус описанной окружности и угол между диагоналями

Найдите квадрат радиуса окружности, описанной вокруг прямоугольника.

Умножьте полученное число на 2, а потом на синус угла между диагоналями.

  • S — искомая площадь прямоугольника;
  • R — радиус описанной окружности;
  • α — любой угол между диагоналями прямоугольника.

Читайте также 🎓❓📐

Как найти периметр зная площадь

Затеяв ремонт, необходимо в первую очередь иметь план действий и рассчитать свой бюджет. Только при грамотной планировке можно добиться качественной работы в короткие сроки. Если вы собираетесь сделать ремонт своего потолка, то необходимо сделать необходимые замеры. Зная площадь потолка можно примерно рассчитать, сколько материалов нужно будет купить и сколько будет стоимость услуги мастеров, если собираетесь обратиться к ним. Но площадь прямоугольника – это еще не все. Иногда бывает так, что нужно знать периметр прямоугольника . встает вопрос можно ли узнать периметр, зная при этом площадь? Рассмотрим этот вопрос повнимательнее, и постараемся найти периметр прямоугольника.

Данные необходимые для того чтобы найти периметр

Сумма всех сторон прямоугольника называется периметром – это еще мы уяснили из курса арифметики начальных классов. Как видно из условия необходимо знать длину сторон. Площадь же – результат умножения двух сторон, в этом случае так же необходимо знать длину сторон. И в первом и во втором случае обязательным условием является знание длин сторон А и В.

Как же через показатель площади найти у прямоугольника периметр? Тут может быть два варианта: первый, если наш прямоугольник с равными сторонами, то есть квадрат, а второй, если длина сторон разная.

При условии, что потолок квадратный то найти периметр очень просто. Зная формулу нахождения площади квадрата, можно выяснить найти длину всех сторон, ведь они у квадрата одинаковые.

  1. Площадь = длина стороны во второй степени. Чтобы найти длину стороны нам нужно переделать данную формулу следующим образом:
  2. Длина стороны =  корень квадратный от площади
  3. Так длина стороны при площади 4 квадратных метров, будет 2 метра, а при 16 квадратных метров 4 метра.
  4. Периметр = длина квадрата умноженная на 4. При длине стороны 2 метра, то периметр будет 8 метров. Тут все просто.

Довольно простой способ, который позволит посчитать периметр квадратного потолка. Квадратный потолок будет отличаться тем что, при большом показателе периметра будет относительно не большие площади. Однако квадратные потолки – это довольно редкий случай. Как правило, такие помещения не очень смотрятся, поэтому наиболее распространенные являются прямоугольные потолки.

Можно ли также найти периметр не квадратного прямоугольника?

Данный способ для прямоугольника с разными сторонами не подходит. Ведь вариантов разности сторон может быть до бесконечности много. И тут для определения периметра обязательным условием является знание хотя бы одной из сторон и площади.

Площадь = длина первой стороны умножается на длину второй стороны

Исходя из этой формулы, зная площадь найти две неизвестные стороны прямоугольника невозможно, но возможно выяснить длину одной стороны, если есть длина первой. Так если площадь прямоугольника 10 квадратных метров, а длина одной из сторон 2 метра, то можно посчитать

10 = 2 умножить на длину неизвестной стороны, следовательно, неизвестная сторона = 10 разделить на 2. Получаем ответ 5 метров.

Периметр = ( 5 + 2 ) * 2. Периметр такого прямоугольника будет 14 метров.

Таким образом, с подсчетом не возникнет проблем, если вы хорошо учили арифметику. Однако для того чтобы упростить себе жизнь, можно обратиться в фирмы по ремонту квартир. Мастера подобных организаций берут на себя весь процесс расчетов и монтажных работ, вам только необходимо будет подписать с ними соответствующие документы и все. Использование подобных услуг – это очень простой способ решения нудной проблемы ремонта потолков. Вы получаете компетентную помощь от высококвалифицированной бригады мастеров, которые имеют большой опыт работы. А подписывая с ними контракт, вы страхуете себя от ненужных проблем, которые порою бывают из-за недопонимания. Договоры о сотрудничестве содержат все нюансы работы, и выполняются в соответствии с законом.

При планировании бюджета на ремонт потолка, после проведенных расчетов необходимо закупить расходные материалы. Рекомендуется покупать немного больше требуемого объема материалов, так как бывают случаи с неожиданным результатом. Так хорошо будет брать запас в 15 процентов – это оптимальный объем. Но еще более приемлемым будет заказать ремонт потолков под ключ, ведь в этом случае нет надобности беспокоиться о закупках. Мастера сами предложат выбрать материалы для ремонта, после того как выбор был сделан они привезут и сделают ремонт. Как правило, у них налажена система логистики, поэтому с доставкой не возникает проблем. Если вы цените свое время и нервы, рекомендуется обратиться к подобным компаниям по ремонту потолков под ключ. Вы получите качественный сервис в короткие сроки, и ваш потолок будет радовать вас как никогда прежде. В любом случае решение остается за вами!

Памятка для учащихся 3-4 классов по математике | Методическая разработка по математике (4 класс) по теме:

Математика

Компоненты арифметических действий и их взаимосвязь.

  1. Компоненты при сложении:

1слагаемое, 2слагаемое, сумма.

  1. Компоненты при вычитании:

уменьшаемое, вычитаемое, разность.

  1. Компоненты при умножении:

1 множитель, 2множитель, произведение.

  1. Компоненты при делении:

делимое, делитель ,частное.

  1. Назвать результаты всех действий:

при сложении — сумма

при вычитании — разность

при умножении — произведение

при делении – частное

  1. Как найти неизвестное слагаемое?

Чтобы найти неизвестное слагаемое, нужно из суммы вычесть известное слагаемое.

Х+4=12                        или                4+х=12

Х=12-4                                        х=12-4                                        

Х=8                                                 х=8

8+4=12                                        4+8=12

  12=12                                        12=12

  1. Как найти неизвестное уменьшаемое?

Чтобы найти неизвестное уменьшаемое, надо к разности прибавить вычитаемое.

Х-7=3

Х=3+7

Х=10

10-7=3

        3=3

  1. Как найти неизвестное вычитаемое?

Чтобы найти неизвестное вычитаемое, надо из уменьшаемого вычесть разность.

8-х =5

х=8-5

х=3

8-3=5

   5=5

9                Как найти неизвестный множитель?        

Чтобы найти неизвестный множитель, надо произведение разделить на известный множитель.

х·3=6                                4·х=8

х=6:3                                х=8:4

х=2                                х=2        

2·3=6                                4·2=8

   6=6                                   8=8

10           Как найти неизвестное делимое?

Чтобы найти неизвестное делимое, надо частное умножить на делитель.

х:5=3

х=3·5

х=15

15:5=3

     3=3

  1. Как найти неизвестный делитель?

Чтобы найти неизвестный делитель, надо делимое разделить на частное.

6:х=2

х =6:2

х=3

6:3=2

   2=2

Геометрический материал.

  1. Что такое квадрат?

Квадрат – это прямоугольник, у которого все стороны равны.

  1. Что такое прямоугольник?

Прямоугольник – это четырёхугольник, у которого все углы прямые. Противоположные стороны прямоугольника  равны.

  1. Что такое треугольник?

Треугольник – многоугольник, у которого три угла и три стороны.

15     Что такое четырёхугольник?

Четырёхугольник – геометрическая фигура, у которой четыре угла и четыре стороны.

  1. Что такое периметр?

Периметр( Ρ) – это сумма длин сторон какой-нибудь геометрической фигуры.

  1. Что такое площадь?

Площадь(S) – это внутренняя часть какой-нибудь геометрической  фигуры

(прямоугольника, квадрата и т.д)

  1. Как найти периметр квадрата?

У квадрата 4 стороны, равные между собой. Чтобы найти сторону квадрата, нужно Ρ□ разделить на 4.

a□ = Ρ□ : 4

  1. Как найти периметр прямоугольника?

Чтобы найти периметр прямоугольника, нужно сложить все 4 стороны   прямоугольника

 Или

сложить длину и ширину прямоугольника и умножить на 2.

        Ρ▬=a+b+a+b

или

    Ρ▬=(a+b)·2

  1. Как найти периметр треугольника?

Чтобы найти периметр треугольника, нужно сложить все 3 стороны.

  1. Как найти сторону квадрата, если известен периметр? 

У квадрата 4 стороны, равные между собой. Чтобы найти сторону квадрата, нужно Ρ разделить на 4.

a=Ρ:4

  1.         Как найти сторону прямоугольника, если известен периметр и другая сторона? 

Чтобы найти сторону прямоугольника, нужно Ρ▬разделить на 2  и  вычесть другую сторону.

a▬=Ρ▬:2 – b

b▬=Ρ▬:2 – a

22        В каких единицах измеряется периметр?

    Периметр измеряется в мм, см, дм, метрах.

23         Как найти площадь квадрата?

Площадь квадрата равна произведению двух его сторон.

S□ =  a · a

24          Как найти площадь прямоугольника?

Чтобы найти площадь прямоугольника, надо длину прямоугольника умножить на его ширину.

S█ =  a · b

25          Как найти сторону прямоугольника, если известна площадь и другая его сторона?

Чтобы найти одну из сторон прямоугольника,  нужно площадь прямоугольника разделить на известную сторону.

a▬=S▬ :  b

b▬= S ▬ : a

   26        В каких единицах измеряется площадь?

   Площадь измеряется в квадратных единицах: мм², см², дм², м².

27      Назвать единицы длины.

Единицы длины — мм, см, дм, м, км.

28           Рассказать таблицу мер длины.

1см  =  10мм

1дм  =  10см

1дм  =  100мм

1м  =    10 дм

1м  =  100 см

1км = 1000м

29       Сколько  квадратных сантиметров

в 1квадратном метре?

1м²  =  10 000см²  

30      Сколько  квадратных  дециметров  

в 1 квадратном  метре?

1м²  =  100дм²  

31            Рассказать  таблицу мер площади.

1м²  =  100дм²  = 10 000см²  

1дм²  = 100см²  =  10 000мм²

1см²  =  100мм²

Масса.

32            Назвать единицы массы.

Масса измеряется в граммах, килограммах, центнерах, тоннах.

33           Рассказать таблицу мер массы.

1кг  = 1000г

1ц  =  100кг

1т  =  10ц

1т  =  1000кг

Время.

        

34        Назвать  единицы измерения времени.

Время  измеряется секундами, минутами, часами, сутками, неделями, месяцами, годами, веками.

35           Рассказать таблицу мер времени.

1мин  =  60сек.

1час  =  60мин

1час  =  3600сек.

1сут.  =  24часа

1год  =  12мес.  =  365сут.  или  366сут.

1век  =  100лет

Взаимосвязь скорости, времени и расстояния.

36     Как  найти  скорость?

Чтобы найти скорость ( v ), надо расстояние ( S ) разделить на время ( t ), затраченное в пути.

v = S : t

37    Как найти время?

Чтобы найти время ( t ), надо расстояние ( S ) разделить на скорость ( v ).

t = S : v

38    Как найти расстояние?

Чтобы найти расстояние ( S ),  нужно скорость ( v ) умножить на время ( t ).

S = v · t

Взаимосвязь цены, количества, стоимости.

39    Что такое цена?

Цена – стоимость одного предмета, единицы товара.

40     Как найти стоимость?

Чтобы найти стоимость, нужно цену умножить на количество.

Ст = Ц · К

41     Как найти цену?

Чтобы найти цену, нужно стоимость разделить на количество.

Ц  =  Ст : К

42   Как найти  количество?

Чтобы найти количество, нужно стоимость разделить на цену.

К = Ст : Ц

Задачи на дроби.

43

Дробь  — ⅔

2 – числитель

3 – знаменатель

 

44  Как найти дробь числа?

Чтобы найти дробь числа, нужно число разделить на знаменатель, а потом умножить на числитель.

45      Как найти число по дроби?

Чтобы  найти число по дроби, нужно число разделить на числитель и умножить на знаменатель.

Взаимосвязь работы, времени и производительности.

46    Что такое производительность?

         Как найти производительность?

Производительностью ( v ) называют работу, выполненную за единицу времени.

Чтобы найти производительность ( v ), надо всю выполненную работу разделить на время.

v  =  A  :  t

47    Как найти выполненную работу?

Выполненная работа равна производительности, умноженной на время работы.

A  =  v  · t

48   Как найти время работы?

Чтобы узнать время работы, надо работу разделить на производительность.

t  =  A  :  v

49   Как  найти среднее арифметическое?

Чтобы найти среднее арифметическое надо сумму разделить на число слагаемых.

Как найти диагональ треугольника зная его стороны. Геометрические фигуры. Прямоугольник. Формулы

Видеокурс «Получи пятерку» включает все темы, необходимые для успешной сдачи ЕГЭ по математике на 60-65 баллов. Полностью все задачи 1-13 Профильного ЕГЭ по математике. Подходит также для сдачи Базового ЕГЭ по математике. Если вы хотите сдать ЕГЭ на 90-100 баллов, вам надо решать часть 1 за 30 минут и без ошибок!

Курс подготовки к ЕГЭ для 10-11 класса, а также для преподавателей. Все необходимое, чтобы решить часть 1 ЕГЭ по математике (первые 12 задач) и задачу 13 (тригонометрия). А это более 70 баллов на ЕГЭ, и без них не обойтись ни стобалльнику, ни гуманитарию.

Вся необходимая теория. Быстрые способы решения, ловушки и секреты ЕГЭ. Разобраны все актуальные задания части 1 из Банка заданий ФИПИ. Курс полностью соответствует требованиям ЕГЭ-2018.

Курс содержит 5 больших тем, по 2,5 часа каждая. Каждая тема дается с нуля, просто и понятно.

Сотни заданий ЕГЭ. Текстовые задачи и теория вероятностей. Простые и легко запоминаемые алгоритмы решения задач. Геометрия. Теория, справочный материал, разбор всех типов заданий ЕГЭ. Стереометрия. Хитрые приемы решения, полезные шпаргалки, развитие пространственного воображения. Тригонометрия с нуля — до задачи 13. Понимание вместо зубрежки. Наглядное объяснение сложных понятий. Алгебра. Корни, степени и логарифмы, функция и производная. База для решения сложных задач 2 части ЕГЭ.

Чтоб правильно рассчитать и выставить диагональ фундамента или опалубки
фундамента — очень хорошо нанять спецов. Но если вы уже несколько раз видели передачу «квадратный метр», несколько раз слышали разговор о том как надо строить, а еще кучу анекдотов о строительстве? — другое дело. Это и дает нам «полное право» предполагать, что мы сами справимся с таким простым делом, как углы и диагонали опалубки фундамента. Именно такого высокого мнения о себе каждый, кто планирует строить баню своими руками (Ха-ха!)

О начале разметки и проектирования фундамента и опалубки я писал в статье . На момент вбивания кольев и установки внешних досок опалубки я уже проверял длину диагонали. Все сходилось до миллиметра. Это самое главное условия получения прямых углов сруба бани. Но после первой разметки были манипуляции с установкой дна ростверка, монтаж внутренних щитов опалубки, доделывание опалубки столбиков от уровня земли до дна будущего фундамента. Конечно же, я очень старался чтоб ничего не сдвинуть, и колья вбивал глубоко.

Но как и во всякой стройке, случился перекосяк. Это не так страшно, как если бы я этого не заметил или я об этом не знал. Поэтому я перед укладкой арматуры решил опять проверить диагонали. Разница получилась в 2 см. Вот и хорошо, что обнаружилось до заливки бетона.

Как вывести диагональ опалубки?

Для упрощения постройки правильной опалубки я делал длину стенок абсолютно равной. Поэтому перекос может получиться только в виде ромба. На рисунке умышлено увеличена степень перекоса опалубки для наглядности.
Для исправления ситуации поступили так:

Такое комбинированное перемещение одной из сторон опалубки (северной на рисунке) не было слишком трудном, поскольку колья и первоначальное расположение опалубки соответствовали правильному положению. Поэтому смещение диагонали было минимальным и усилия по «корректировке» положения щитов не вызывали механического напряжения и усилий.

Способ установки углов по равным диагоналям можно использовать только при условии равенства сторон. Равенства диагоналей
будет достаточно!

Для сторон опалубки с большим размером возможно применить правило «золотого» треугольника. Если такой треугольник, согласно теореме Пифагора, имеет стороны 3, 4, то гипотенуза равна 5 единицам. Таким образом, достаточно отмерить на сторонах опалубки части кратные 3 и 4 у вершины прямого угла и тогда расстояние между контрольными точками будет 5 частей! Это и будет гарантией прямых углов и равенства диагоналей!

Для осуществления правильного планирования монтажа опалубки
очень рекомендую использовать метод обноски, который позволяет в любое время монтажных работ производить сверку углов, снимать и повторно устанавливать шнуры периметра фундамента.

Перед заливкой фундамента не поленитесь еще раз проверить диагонали. Это лишним не будет! Бетон невозможно легко и быстро поправить. Ошибки исправлять очень дорого и долго. Фундамент для сруба имеет больше требований к качеству чем фундамент для каменного дома. Раствором уже ничего не выровнять!

Не забудьте перед заливкой для ее легкого демонтажа!

Определение.

Прямоугольник
— это четырехугольник у которого две противоположные стороны равны и все четыре угла одинаковы.

Прямоугольники отличаются между собой только отношением длинной стороны к короткой, но все четыре угла у них прямые, то есть по 90 градусов.

Длинную сторону прямоугольника называют длиной прямоугольника
, а короткую — шириной прямоугольника
.

Стороны прямоугольника одновременно является его высотами.

Основные свойства прямоугольника

Прямоугольником могут быть параллелограмм, квадрат или ромб.

1. Противоположные стороны прямоугольника имеют одинаковую длину, то есть они равны:

AB = CD, BC = AD

2. Противоположные стороны прямоугольника параллельны:

3. Прилегающие стороны прямоугольника всегда перпендикулярны:

AB ┴ BC, BC ┴ CD, CD ┴ AD, AD ┴ AB

4. Все четыре угла прямоугольника прямые:

∠ABC = ∠BCD = ∠CDA = ∠DAB = 90°

5. Сумма углов прямоугольника равна 360 градусов:

∠ABC + ∠BCD + ∠CDA + ∠DAB = 360°

6. Диагонали прямоугольника имеют одинаковой длины:

7. Сумма квадратов диагонали прямоугольника равны сумме квадратов сторон:

2d
2 = 2a
2 + 2b
2

8. Каждая диагональ прямоугольника делит прямоугольник на две одинаковые фигуры, а именно на прямоугольные треугольники.

9. Диагонали прямоугольника пересекаются и в точке пересечения делятся пополам:

10. Точка пересечения диагоналей называется центром прямоугольника и также является центром описанной окружности

11. Диагональ прямоугольника является диаметром описанной окружности

12. Вокруг прямоугольника всегда можно описать окружность, так как сумма противоположных углов равна 180 градусов:

∠ABC = ∠CDA = 180° ∠BCD = ∠DAB = 180°

13. В прямоугольник, у которого длина не равна ширине, нельзя вписать окружность, так как суммы противоположных сторон не равны между собой (вписать окружность можно только в частный случай прямоугольника — квадрат).

Стороны прямоугольника

Определение.

Длиной прямоугольника
называют длину более длинной пары его сторон. Шириной прямоугольника
называют длину более короткой пары его сторон.

Формулы определения длин сторон прямоугольника

1. Формула стороны прямоугольника (длины и ширины прямоугольника) через диагональ и другую сторону:

a
= √d
2 — b
2

b
= √d
2 — a
2

2. Формула стороны прямоугольника (длины и ширины прямоугольника) через площадь и другую сторону:

Диагональ прямоугольника

Определение.

Диагональю прямоугольника
называется любой отрезок соединяющий две вершины противоположных углов прямоугольника.

Формулы определения длины диагонали прямоугольника

1. Формула диагонали прямоугольника через две стороны прямоугольника (через теорему Пифагора):

d
= √a
2 + b
2

2. Формула диагонали прямоугольника через площадь и любую сторону:

4. Формула диагонали прямоугольника через радиус описанной окружности:

d
= 2R

5. Формула диагонали прямоугольника через диаметр описанной окружности:

d
= D о

6. Формула диагонали прямоугольника через синус угла, прилегающего к диагонали, и длину стороны противоположной этому углу:

8. Формула диагонали прямоугольника через синус острого угла между диагоналями и площадью прямоугольника

d
= √2S: sin β

Периметр прямоугольника

Определение.

Периметром прямоугольника
называется сумма длин всех сторон прямоугольника.

Формулы определения длины периметру прямоугольника

1. Формула периметру прямоугольника через две стороны прямоугольника:

P = 2a
+ 2b

P = 2(a
+ b
)

2. Формула периметру прямоугольника через площадь и любую сторону:

P =2S + 2a
2
=2S + 2b
2
a

b

3. Формула периметру прямоугольника через диагональ и любую сторону:

P = 2(a
+ √d
2 — a
2
) = 2(b
+ √d
2 — b
2
)

4. Формула периметру прямоугольника через радиус описанной окружности и любую сторону:

P = 2(a
+ √4R 2 — a
2
) = 2(b
+ √4R 2 — b
2
)

5. Формула периметру прямоугольника через диаметр описанной окружности и любую сторону:

P = 2(a
+ √D o 2 — a
2
) = 2(b
+ √D o 2 — b
2
)

Площадь прямоугольника

Определение.

Площадью прямоугольника
называется пространство ограниченный сторонами прямоугольника, то есть в пределах периметра прямоугольника.

Формулы определения площади прямоугольника

1. Формула площади прямоугольника через две стороны:

S = a · b

2. Формула площади прямоугольника через периметр и любую сторону:

5. Формула площади прямоугольника через радиус описанной окружности и любую сторону:

S = a
√4R 2 — a
2
= b
√4R 2 — b
2

6. Формула площади прямоугольника через диаметр описанной окружности и любую сторону:

S = a
√D o 2 — a
2
= b
√D o 2 — b
2

Окружность описанная вокруг прямоугольника

Определение.

Окружностью описанной вокруг прямоугольника
называется круг проходящий через четыре вершины прямоугольника, центр которого лежит на пересечении диагоналей прямоугольника.

Формулы определения радиуса окружности описанной вокруг прямоугольника

1. Формула радиуса окружности описанной вокруг прямоугольника через две стороны:

Инструкция

Определим длину диагонали прямоугольника со сторонами 3 и 4 см.

Находим сумму квадратов сторон прямоугольника 32 + 42 = 9 + 16 = 25.

Извлечь из полученного результата квадратный корень – длина диагонали 5 см.

Видео по теме

Обратите внимание

Диагонали прямоугольника равны. Если найдена длина одной, то длина второй будет абсолютно такой же.

Источники:

  • как найти длину диагонали в прямоугольнике

Квадрат – красивая и простая плоская геометрическая фигура. Это прямоугольник с равными сторонами. Как же найти диагональ
квадрата
, если известна длина его стороны?

Инструкция

длина диагонали квадрата
равна длине его стороны умноженной на из двух.

Видео по теме

Полезный совет

Если точность математического результата не очень важна, то вместо корня из двух можно использовать его приблизительное значение 1,41.

Параллелограмм — это четырехугольник, противоположные стороны которого параллельны. Прямые, соединяющие его противоположные углы, называются диагоналями. Их длина зависит не только от длин сторон фигуры, но и от величин углов в вершинах этого многоугольника, поэтому без знания хотя бы одного из углов вычислить длины диагоналей можно только в исключительных случаях. Таковыми являются частные случаи параллелограмма — квадрат и прямоугольник.

Инструкция

Если длины всех сторон параллелограмма одинаковы (a), то эту фигуру можно назвать еще и квадратом. Величины всех его углов 90°, а длины диагоналей (L) одинаковы и могут быть рассчитаны по теореме Пифагора для прямоугольного треугольника. Умножьте длину стороны на корень из двойки — результат и будет длиной каждой из его диагоналей: L=a*√2.

Если о параллелограмме известно, что он прямоугольником с указанными в длиной (a) и шириной (b), то и в этом случае длины диагоналей (L) будут равны. И здесь тоже задействуйте теорему Пифагора для треугольника, в котором гипотенузой является диагональ, а катетами — две смежные стороны четырехугольника. Искомую величину рассчитайте извлечением корня из возведенных в квадрат и прямоугольника: L=√(a²+b²).

Для всех остальных случаев знания одних только длин сторон хватит лишь для величины, включающей в себя длины сразу обеих диагоналей — сумма их квадратов по определению равна удвоенной сумме квадратов длин сторон. Если же в к длинам двух смежных сторон параллелограмма (a и b) известен еще и угол между ними (γ), то это позволит рассчитать длины каждого отрезка, соединяющего противоположные углы . Длину диагонали (L₁), лежащей напротив известного угла, найдите по теореме косинусов — сложите квадраты длин смежных сторон, от результата отнимите произведение этих же длин на косинус угла между ними, а из полученной величины извлеките квадратный корень: L₁ = √(a²+b²-2*a*b*cos(γ)). Для нахождения длины другой диагонали (L₂) можно воспользоваться свойством параллелограмма, приведенным в начале этого шага — удвойте сумму квадратов длин двух сторон, от результата отнимите квадрат уже рассчитанной диагонали, а из полученного значения извлеките корень. В общем виде эту формулу можно записать так: L₂ = √(a²+b²- L₁²) = √(a²+b²-(a²+b²-2*a*b*cos(γ))) = √(a²+b²-a²-b²+2*a*b*cos(γ)) = √(2*a*b*cos(γ)).

Источники:

  • как найти длину диагонали параллелограмма

Можно назвать параллелограмм, диагонали которого делят пополам углы, лежащие в вершинах фигуры. Кроме этого свойства диагонали ромба
примечательны тем, что являются осями симметрии многоугольника, пересекаются только под прямым углом, а единственная общая точка делит каждую из них на два равных отрезка. Эти свойства позволяют легко рассчитать длину одной из диагоналей, если известна длина другой и еще какой-нибудь параметр фигуры — размер стороны, угол в одной из вершин, площадь и т.д.

Инструкция

Если кроме длины одной из (l) о рассматриваемом четырехугольнике известно, что он частным случаем ромба
— квадратом, никаких расчетов производить не придется. В этом случае длины обеих диагоналей — просто приравняйте искомую величину (L) к известной: L=l.

Знание длины стороны ромба
(a) в дополнение к длине одной из диагоналей (l) позволит длину другой (L) по теореме Пифагора. Это потому, что две половины пересекающихся диагоналей образуют со стороной ромба
прямоугольный треугольник. Половины диагоналей в нем являются катетами, а сторона — гипотенузой, поэтому равенство, вытекающее из теоремы Пифагора записать так: a² = (l/2)² + (L/2)². Для использования в расчетах преобразуйте его к такому виду: L = √(4*a²-l²).

При известной величине одного из углов (α) ромба
и длине одной из диагоналей (l) для нахождения величины другой (L) рассмотрите тот же прямоугольный треугольник. Тангенс половины известного угла в нем отношению длины противолежащего катета — половины диагонали l — к прилежащему — половине диагонали L: tg(α/2) = (l/2)/(L/2) = l/L. Поэтому для искомой величины используйте формулу L = l/tg(α/2).

Если в условиях задачи приведена длина периметра (P) ромба
и размер его диагонали (l), формулу вычисления длины второй (L) можно свести к равенству, использованному во втором шаге. Для этого разделите периметр на четверку и замените этим выражением длину стороны в : L = √(4*(P/4)²-l²) = √(P²/4-l²).

В исходных условиях кроме длины одной из диагоналей (l) может быть приведена и площадь (S) фигуры. Тогда для вычисления длины второй диагонали ромба
(L) используйте очень простой алгоритм — удвойте площадь и разделите полученное значение на длину известной диагонали: L = 2*S/l.

Прямоугольник
— это четырехугольник, у которого каждый угол является прямым.

Доказательство

Свойство объясняется действием признака 3 параллелограмма (то есть \angle A = \angle C
, \angle B = \angle D
)

2. Противоположные стороны равны.

AB = CD,\enspace BC = AD

3. Противоположные стороны параллельны.

AB \parallel CD,\enspace BC \parallel AD

4. Прилегающие стороны перпендикулярны друг другу.

AB \perp BC,\enspace BC \perp CD,\enspace CD \perp AD,\enspace AD \perp AB

5. Диагонали прямоугольника равны.{\circ}

14. Прямоугольник может содержать вписанную окружность и только одну, если он имеет одинаковые длины сторон (является квадратом).

Как найти площадь квадрата если известен периметр

Калькулятор вычислит площадь квадрата по периметру! 1. Заполните значение периметра квадрата. 2. Мы вычислим площадь и сторону квадрата. 3. Представим подробное решение письменно и с комментариями и объяснениями!

Вычисли площадь квадрата периметр которого равен 24!

Некоторых такой вопрос ставит в тупик, но достаточно вспомнить, что квадрат равносторонняя фигура и у него 4 ре стороны. И сразу всё становится очень просто!

1. Вы не поняли? Тогда разберём этот пример как узнать площадь квадрата зная периметр.

Пример: вычислить площадь квадрата периметр которого равен 24. Для начала разделим на 4-ре значение периметра 24 и получим 6 см или дм, единицы вычисления используются те, которые были в задаче.

После это го умножим полученное значение стороны квадрата на саму себя, или возведём во вторую степень. 6 х 6 = 36 см2.

Это и будет искомое значение площадь квадрата с периметром 24, сначала мы вычислили сторону равную 6 и затем нашли площадь равную 36.

Как найти площадь квадрата по периметру?

Просто используй наш онлайн калькулятор квадрата и вычисли площадь квадрата периметр которого равен абсолютно любому числу, 12, 16, 360 хоть 81 см или дм . Любое решение будет расписано с комментариями и двумя вариантами написания, тебе останется только переписать и использовать полученное значение.

Периметр квадрата, да и любой геометрической фигуры, равен сумме длин всех сторон.

Квадрат – правильный четырехугольник, то есть четыре его стороны имеют равную длину и образуют

четыре прямых угла.

Квадрат является боковой гранью куба.

Найти периметр квадрата крайне просто. Для этого просто следуйте этим шагам.

Метод 1 из 3: Найдите периметр квадрата, если известна длина одной стороны

Умножьте длину стороны на 4. Квадрат имеет четыре стороны одинаковой длины, значит для нахождения

периметра квадрата нужно просто умножить длину любой из сторон на 4.

Следуйте простой формуле:

где P = периметр и S = длина стороны.

Пример: Если одна из сторон квадрата 5, то P = 4 * 5 или 20.

Метод 2 из 3: Найдите периметр квадрата, если известна его площадь

Для этого нужно понимать формулу нахождения площади квадрата

или длина умноженная на ширину.

Так как все стороны квадрата равны, то формулу можно упростить до:

или квадрат длины стороны.

Например, если одна из сторон – 4, то площадь – 4 2 или 4 * 4, что дает нам 16.

Метод 3 из 3: Найдите периметр квадрата, если известна его диагональ

Периметр квадрата равен произведению длины его диагонали на два корня из двух.

Квадрат представляет собой положительный четырехугольник (либо ромб), в котором все углы являются прямыми, а стороны равны между собой. Как и у всякого другого верного многоугольника, у квадрата дозволено высчитать периметр и площадь. Если площадь квадрата теснее знаменита, то обнаружить его стороны, а после этого и периметр не составит труда.

Инструкция

1. Площадь квадрата находится по формуле:S = a?Это обозначает, что для того, дабы вычислить площадь квадрата , надобно умножить длины 2-х его сторон друг на друга. Как следствие, если знать площадь квадрата , то при извлечении корня из данного значения дозволено узнать длину стороны квадрата .Пример: площадь квадрата 36 см?, дабы узнать сторону данного квадрата , нужно извлечь квадратный корень из значения площади. Таким образом, длина стороны данного квадрата 6 см

2. Для нахождения периметр а квадрата нужно сложить длины всех его сторон. С подмогой формулы это дозволено выразить так:P = a+a+a+a.Если извлечь корень из значения площади квадрата , а после этого сложить получившуюся величину 4 раза, то дозволено обнаружить периметр квадрата .

3. Пример: Дан квадрат с площадью 49 см?. Требуется обнаружить его периметр .Решение:Вначале нужно извлечь корень площади квадрата : ?49 = 7 смЗатем, вычислив длину стороны квадрата , дозволено вычислить и периметр : 7+7+7+7 = 28 смОтвет: периметр квадрата площадью 49 см? составляет 28 см

Зачастую в геометрических задачах требуется обнаружить длину стороны квадрата, если вестимы другие его параметры – такие, как площадь, диагональ либо периметр.

Вам понадобится

Инструкция

1. Если вестима площадь квадрата, то для того, дабы обнаружить сторону квадрата, нужно извлечь квадратный корень из числового значения площади (потому что площадь квадрата равняется квадрату его стороны):a=?S, гдеa – длина стороны квадрата;S – площадь квадрата.Единицей измерения стороны квадрата будет являться линейная единица измерения длины, соответствующая единице измерения площади. Скажем, если площадь квадрата дана в сантиметрах квадратных, то длина его стороны получится примитивно в сантиметрах.Пример:Площадь квадрата составляет 9 квадратных метров.Обнаружить длину стороны квадрата.Решение:a=?9=3Ответ:Сторона квадрата равняется 3 метрам.

2. В том случае, когда знаменит периметр квадрата, для определения длины стороны необходимо числовое значение периметра поделить на четыре (потому что квадрат имеет четыре стороны идентичной длины):a=P/4, где:a – длина стороны квадрата;P – периметр квадрата.Единицей измерения стороны квадрата будет являться та же самая линейная единица измерения длины как и у периметра.2), где:a – длина стороны квадрата;d – длина диагонали квадрата.Единицей измерения стороны квадрата будет являться единица измерения длины та же самая, что и у диагонали. Скажем, если диагональ квадрата измерена в сантиметрах, то и длина его стороны получится в сантиметрах.Пример:Диагональ квадрата равняется 10 метров.Обнаружить длину стороны квадрата.Решение:a=10/?2, либо примерно: 7,071Ответ:Длина стороны квадрата равняется 10/?2, либо приблизительно 1,071 метра.

Квадрат – прекрасная и простая плоская геометрическая фигура. Это прямоугольник с равными сторонами. Как же обнаружить периметр квадрата , если знаменита длина его стороны?

Инструкция

1. Раньше каждого, стоит припомнить, что периметр есть ни что иное как сумма длин сторон геометрической фигуры. Рассматриваемый нами квадрат имеет четыре стороны. Больше того, по определению квадрата , все эти стороны равны между собой.Из этих предпосылок вытекает простая формула для нахождения периметр а квадратапериметр квадрата равен длине стороны квадрата , умноженной на четыре:Р = 4а, где а – длина стороны квадрата .

Видео по теме

Периметром называют всеобщую длину границы фигуры почаще каждого на плоскости. Квадрат — положительный четырехугольник либо ромб, у которого все углы прямые, либо параллелограмм, у которого все стороны и углы равны.

Вам понадобится

Инструкция

1. Периметр квадрата равен сумме длин его сторон. Потому что квадрат, по своей сути, есть четырехугольник, то и сторон у него четыре, а значит периметр равен сумме длин четырех сторон либо P = a+b+c+d.

2. Квадрат, как видно из определения, верная геометрическая фигура, а это значит, что все его стороны равны. Значит a=b=c=d. Следственно P = a+a+a+a либо P = 4*a.

3. Пускай сторона квадрата равна 4, то есть a=3. Тогда периметр либо длина квадрата , по полученной формуле, будет равен P = 4*3 либо P=12. Число 12 и будет являться длиной либо, что одно и тоже, периметром квадрата .

Видео по теме

Обратите внимание!
Периметр квадрата величина неизменно правильная, как и любая иная длина.

Полезный совет
Аналогичным образом дозволено обнаружить и периметр ромба, потому что квадрат является частным случаем ромба с прямыми углами.

Периметр характеризует длину замкнутого силуэта. Как и площадь, он может быть обнаружен по иным величинам, приведенным в условии задачи. Задачи на нахождении периметра крайне зачастую встречаются в школьном курсе математики.

Инструкция

1. Зная периметр и сторону фигуры, дозволено обнаружить иную ее сторону, а также площадь. Сам же периметр, в свою очередь, может быть обнаружен по нескольким заданным сторонам либо по углу и сторонам, в зависимости от условий задачи. Также в ряде случаев его выражают через площадь. Особенно примитивно находится периметр прямоугольника. Начертите прямоугольник с одной из сторон, равной а, и диагональю, равной d. Зная эти две величины, обнаружьте по теореме Пифагора иную его сторону, которая является шириной прямоугольника. Обнаружив ширину прямоугольника, вычислите его периметр дальнейшим образом: p=2(a+b). Эта формула объективна для всех прямоугольников, от того что у всякого из них четыре стороны.

2. Обратите внимание на тот факт, что периметр треугольника в большинстве задач находят при наличии информации правда бы об одном его угле. Впрочем, имеются и задачи, в которых все стороны треугольник знамениты, и тогда периметр может быть вычислен простым суммированием, без применения тригонометрических вычислений: p=a+b+c, где a, b и c – стороны. Но такие задачи встречаются в учебниках редко, от того что метод их решения явствен. Больше трудные задачи по нахождению периметра треугольника решайте поэтапно. Скажем, начертите равнобедренный треугольник, у которого знамениты основание и угол при нем. Для того дабы обнаружить его периметр, сначала обнаружьте стороны a и b дальнейшим образом: b=c/2cos?. От того что a=b (треугольник равнобедренный), сделайте дальнейший итог: a=b=c/2cos?.

3. Периметр многоугольника вычисляйте аналогичным образом, складывая длины всех его сторон: p=a+b+c+d+e+f и так дальше. Если многоугольник положительный и вписан в окружность либо описан около нее, вычислите длину одной из его сторон, а после этого умножьте на их число. Скажем, дабы обнаружить стороны шестиугольника, вписанного в окружность, действуйте дальнейшим образом: a=R, где a – сторона шестиугольника, равная радиусу описанной окружности. Соответственно, если шестиугольник верный, то его периметр равен: p=6a=6R. Если окружность вписана в шестиугольник, то сторона последнего равна: a=2r?3/3. Соответственно, периметр такой фигуры обнаружьте дальнейшим образом: p=12r?3/3.

Хоть слово «периметр» и случилось от греческого обозначения окружности, им принято называть суммарную длину границ всякий плоской геометрической фигуры, включая квадрат. Вычисление этого параметра, как водится, трудности не представляет и может быть осуществлено несколькими методами, в зависимости от знаменитых начальных данных.

Инструкция

1. Если вестима длина стороны квадрата (t), то для нахождения его периметра (p) примитивно увеличьте эту величину в четыре раза: p=4*t.

2. Если длина стороны незнакома, но в условиях задачи дана длина диагонали (c), то этого довольно для вычисления длины сторон, а следственно и периметра (p) многоугольника. Используйте теорему Пифагора, которая заявляет, что квадрат длины длинной стороны прямоугольного треугольника (гипотенузы) равен сумме квадратов длин коротких сторон (катетов). В прямоугольном треугольнике, составленном из 2-х смежных сторон квадрата и соединяющего их крайние точки отрезка, гипотенуза совпадает с диагональю четырехугольника. Из этого вытекает, что длина стороны квадрата равна отношению длины диагонали к квадратному корню из двойки. Используйте это выражение в формуле для вычисления периметра из предыдущего шага: p=4*c/?2.

3. Если дана лишь площадь (S) ограниченного периметром квадрата участка плоскости, то и этого будет довольно, дабы определить длину одной стороны. Потому что площадь всякого прямоугольника равна произведению длин его смежных сторон, то для нахождения периметра (p) извлеките квадратный корень из площади, а итог увеличьте в четыре раза: p=4*?S.

4. Если знаменит радиус описанной вблизи квадрата окружности (R), то для нахождения периметра многоугольника (p) умножьте его на восемь и поделите полученный итог на квадратный корень из двойки: p=8*R/?2.

5. Если окружность, радиус которой вестим, вписана в квадрат, то вычисляйте его периметр (p) простым умножением радиуса (r) на восьмерку: P=8*r.

6. Если рассматриваемый квадрат в условиях задачи описан координатами своих вершин, то для вычисления периметра вам потребуются данные лишь о 2-х вершинах, принадлежащих к одной из сторон фигуры. Определите длину этой стороны, исходя из все той-же теоремы Пифагора для треугольника, составленного из нее самой и ее проекций на оси координат, а полученный итог увеличьте в четыре раза. Потому что длины проекций на координатные оси равны модулю разностей соответствующих координат 2-х точек (X?;Y? и X?;Y?), то формулу дозволено записать так: p=4*?((X?-X?)?+(Y?-Y?)?).

Периметром в всеобщем случае называют длину линии, которая ограничивает замкнутую фигуру. Для многоугольников периметром является сумма всех длин сторон. Эту величину дозволено измерить, а для многих фигур и легко рассчитать, если вестимы длины соответствующих элементов.

Вам понадобится

  • – линейка либо рулетка;
  • – крепкая нить;
  • – роликовый дальномер.

Инструкция

1. Дабы измерить периметр произвольного многоугольника, измерьте при помощи линейки либо иным измерительным прибором все его стороны, а после этого обнаружьте их сумму. Если дан четырехугольник со сторонами 5, 3, 7 и 4 см, которые измерены линейкой, обнаружьте периметр, сложив их совместно Р=5+3+7+4=19 см.

2. Если же фигура произвольная и включает в себя не только прямые линии, то измерьте ее периметр традиционной веревкой либо ниткой. Для этого расположите ее так, дабы она верно повторяла все линии, ограничивающие фигуру, и сделайте на ней отметку, если дозволено, примитивно обрежьте ее дабы избежать путаницы. После этого при помощи рулетки либо линейки, измерьте длину нитки, она и будет равна периметру данной фигуры. Непременно следите за тем, дабы нить максимально верно повторяла линию для большей точности итога.

3. Периметр трудной геометрической фигуры измеряйте роликовым дальномером (курвиметром). Для этого не линии намечается точка, в которую устанавливается ролик дальномера и прокатывается по ней, до возвращения в начальную точку. Дистанция, измеренная роликовым дальномером, и будет равна периметру фигуры.

4. Периметр некоторых геометрических фигур вычисляйте. Скажем, дабы обнаружить периметр всякого положительного многоугольника (выпуклого многоугольника, стороны которого равны), длину стороны умножьте на число углов либо сторон (они равны). Дабы обнаружить периметр верного треугольника со стороной 4 см умножьте это число на 3 (Р=4?3=12 см).

5. Дабы обнаружить периметр произвольного треугольника, сложите длины всех его сторон. Если не даны все стороны, а есть углы между ними, обнаружьте их по теореме синуса либо косинуса. Если знамениты две стороны прямоугольного треугольника, третью обнаружьте по теореме Пифагора и обнаружьте их сумму. Скажем, если знаменито, что катеты прямоугольного треугольника равны 3 и 4 см, то гипотенуза будет равна ?(3?+4?)=5 см. Тогда периметр Р=3+4+5=12 см.

6. Дабы обнаружить периметр круга, обнаружьте длину окружности, которая его ограничивает. Для этого ее радиус r умножьте на число ??3,14 и число 2 (P=L=2. r). Если знаменит диаметр, рассматривайте, что он равен двум радиусам.

Периметром многоугольника называют замкнутую ломаную линию, составленную из всех его сторон. Нахождение длины этого параметра сводится к суммированию длин сторон. Если все отрезки, образующие периметр такой двухмерной геометрической фигуры, имеют идентичные размеры, многоугольник именуется верным. В этом случае вычисление периметра гораздо упрощается.

Инструкция

1. В самом простом случае, когда вестимы длина стороны (а) верного многоугольника и число вершин (n) в нем, для вычисления длины периметра (Р) примитивно перемножьте эти две величины: Р = а*n. Скажем, длина периметра верного шестиугольника со стороной в 15 см должна быть равна 15*6=90 см.

2. Вычислить периметр такого многоугольника по вестимому радиусу (R) описанной около него окружности тоже допустимо. Для этого придется вначале выразить длину стороны с применением радиуса и числа вершин (n), а после этого умножить полученную величину на число сторон. Дабы рассчитать длину стороны умножьте радиус на синус числа Пи, поделенного на число вершин, а итог удвойте: R*sin(?/n)*2. Если вам комфортнее вычислять тригонометрическую функцию в градусах, замените число Пи на 180°: R*sin(180°/n)*2. Периметр вычислите умножением полученной величины на число вершин: Р = R*sin(?/n)*2*n = R*sin(180°/n)*2*n. Скажем, если шестиугольник вписан в круг с радиусом 50 см, его периметр будет иметь длину 50*sin(180°/6)*2*6 = 50*0,5*12 = 300 см.

3. Схожим методом дозволено посчитать периметр, не зная длины стороны положительного многоугольника , если он описан около окружности с знаменитым радиусом (r). В этом случае формула для вычисления размера стороны фигуры будет отличаться от предыдущей лишь задействованной тригонометрической функцией. Замените в формуле синус на тангенс, дабы получить такое выражение: r*tg(?/n)*2. Либо для расчетов в градусах: r*tg(180°/n)*2. Для вычисления периметра увеличьте полученную величину в число раз, равное числу вершин многоугольника : Р = r*tg(?/n)*2*n = r*tg(180°/n)*2*n. Скажем, периметр восьмиугольника, описанного вблизи круга с радиусом в 40 см, будет примерно равен 40*tg(180°/8)*2*8 ? 40*0,414*16 = 264,96 см.

Квадрат представляет собой геометрическую фигуру, состоящую из четырех сторон идентичной длины и четырех прямых углов, всякий из которых равен 90°. Определение площади либо периметра четырехугольника, причем всякого, требуется не только при решении задач по геометрии, но и в повседневной жизни. Эти знания могут стать пригодными, скажем, во время ремонта при расчете необходимого числа материалов – покрытий для пола, стен либо потолка, а также для разбивки газонов и грядок и т.д.

Инструкция

1. Для определения площади квадрата умножьте величину длины на величину ширины. Потому что в квадрате длина и ширина идентичны, то значение одной стороны довольно построить в квадрат. Таким образом, площадь квадрата равна длине его стороны, возведенной в квадрат. Единицей измерения площади могут быть квадратные миллиметры, сантиметры, дециметры, метры, километры.Дабы определить площадь квадрата, дозволено воспользоваться формулойS = aa, где S – площадь квадрата,а – сторона квадрата.

2. Пример № 1. Комната имеет форму квадрата. Сколько ламината (в кв.м) понадобится для того, дабы всецело покрыть пол, если длина одной стороны комнаты составляет 5 метров.Запишите формулу: S = aa. Подставьте в нее указанные в условии данные.Потому что а = 5 м, следственно, площадь будет равнаS (комнаты) = 5х5= 25 кв.м, значит, и S (ламината) = 25 кв.м.

3. Периметр представляет собой всеобщую длину границы фигуры. В квадрате периметр – это длина всех четырех, причем идентичных, сторон. То есть, периметр квадрата представляет собой сумму всех его четырех сторон. Дабы вычислить периметр квадрата, довольно знать длину одной его стороны. Измеряется периметр в миллиметрах, сантиметрах, дециметрах, метрах, километрах.Для определения периметра имеется формула:P = a + а + а + а илиP = 4a, гдеР – периметр,а – длина стороны.

4. Пример № 2. Для отделочных работ помещения в форме квадрата требуются потолочные плинтуса. Вычислите всеобщую длину (периметр) плинтусов, если величина одной стороны комнаты равна 6 метров. Запишите формулу P = 4a.Подставьте в нее указанные в условии данные:Р (комнаты) = 4 х 6 = 24 метра.Следственно, длина потолочных плинтусов тоже будет равна 24 метров.

Видео по теме

Обратите внимание!
Для квадрата объективны следующие определения:Квадрат – это прямоугольник, тот, что владеет равными между собой сторонами.Квадрат – это специальная разновидность ромба, у которого весь из углов равен 90 градусам.Являясь положительным четырехугольником, вокруг квадрата дозволено описать либо вписать окружность. Радиус вписанной в квадрат окружность дозволено обнаружить по формуле:R = t/2, где t – сторона квадрата.Если же окружность описана вокруг него, то ее радиус находится так:R = (?2*t)/2Исходя из данных формул, дозволено вывести новые для нахождения периметра квадрата:P = 8*R, где R – радиус вписанной окружности;P = 4*?2*R, где R – радиус описанной окружности.Квадрат является уникальной геометрической фигурой, от того что он безусловно симметричен, самостоятельно от того, как и где провести ось симметрии.

3 класс, периметр и площадь прямоугольника

Дата публикации: .

Что такое прямоугольник и квадрат

Прямоугольник – это четырёхугольник, у которого все углы прямые. Значит, противоположные стороны равны друг другу.

Квадрат – это прямоугольник, у которого равны и стороны, и углы. Его называют правильным четырёхугольником.

Четырёхугольники, в том числе прямоугольники и квадраты, обозначаются 4 буквами – вершинами. Для обозначения вершин используют латинские буквы: A, B, C, D

Пример.
Читается так: четырёхугольник ABCD; квадрат EFGH.

Что такое периметр прямоугольника? Формула расчета периметра

Периметр прямоугольника – это сумма длин всех сторон прямоугольника или сумма длины и ширины, умноженная на 2.

Периметр обозначается латинской буквой P. Так как периметр – это длина всех сторон прямоугольника, то он периметр записывается в единицах длины: мм, см, м, дм, км.

Например, периметр прямоугольника АВСD обозначается как PABCD, где А, В, С, D – это вершины прямоугольника.

Запишем формулу периметра четырехугольника ABCD:

PABCD = AB + BC + CD + AD = 2 * AB + 2 * BC = 2 * (AB + BC)

Пример.
Задан прямоугольник ABCD со сторонами: AB=СD=5 см и AD=BC=3 см.
Определим PABCD.

Решение:
1. Нарисуем прямоугольник ABCD с исходными данными.
2. Напишем формулу для расчета периметра данного прямоугольника:

PABCD = 2 * (AB + BС)

3. Подставим в формулу наши данные:

PABCD = 2 * (5 см + 3 см) = 2 * 8 см = 16 см

Ответ: PABCD = 16 см.

Формула расчета периметра квадрата

У нас есть формула для определения периметра прямоугольника.

PABCD = 2 * (AB + BC)

Применим её для определения периметра квадрата. Учитывая, что все стороны квадрата равны, получаем:

PABCD= 4 * AB

Пример.
Задан квадрат ABCD со стороной, равной 6 см. Определим периметр квадрата.

Решение.
1. Нарисуем квадрат ABCD с исходными данными.

2. Вспомним формулу расчета периметра квадрата:

PABCD = 4 * AB

3. Подставим в формулу наши данные:

PABCD = 4 * 6 см = 24 см

Ответ: PABCD = 24 см.

Задачи на нахождение периметра прямоугольника

1. Измерь ширину и длину прямоугольников. Определи их периметр.

2. Нарисуй прямоугольник ABCD со сторонами 4 см и 6 см. Определи периметр прямоугольника.

3. Нарисуй квадрат СEOM со стороной 5 см. Определи периметр квадрата.

Где используется расчет периметра прямоугольника?

1. Задан участок земли, его нужно обнести забором. Какой длины будет забор?

В данной задаче необходимо точно рассчитать периметр участка, чтобы не купить лишний материал для постройки забора.

2. Родители решили сделать ремонт в детской комнате. Необходимо знать периметр комнаты и её площадь, чтобы правильно рассчитать количество обоев.
Определи длину и ширину комнаты, в которой ты живешь. Определи периметр своей комнаты.

Что такое площадь прямоугольника?

Площадь – это числовая характеристика фигуры.
Площадь измеряется квадратными единицами длины: см2, м2, дм2 и др. (сантиметр в квадрате, метр в квадрате, дециметр в квадрате и т.д.)
В вычислениях обозначается латинской буквой S.

Для определения площади прямоугольника необходимо длину прямоугольника умножить на его ширину.
Площадь прямоугольника вычисляется умножением длины АК на ширину КМ. Запишем это в виде формулы.

S AKMO = AK * KM

Пример.
Чему равна площадь прямоугольника AKMO, если его стороны равны 7 см и 2 см?

S AKMO= AK * KM = 7 см * 2 см = 14 см2.

Ответ: 14 см2.

Формула вычисления площади квадрата

Площадь квадрата можно определить, умножив сторону саму на себя.

Пример.
В данном примере площадь квадрата вычисляется умножением стороны АB на ширину BC, но так как они равны, получается умножение стороны AB на AB.

S AВСО = AB * BC = AB * AB

Пример.
Определи площадь квадрата AKMO со стороной 8 см.

S AKMО = AK * KM = 8 см * 8 см = 64 см2

Ответ: 64 см2.

Задачи на нахождение площади прямоугольника и квадрата

1.Задан прямоугольник со сторонами 20 мм и 60 мм. Вычисли его площадь. Запиши ответ в квадратных сантиметрах.

2. Был куплен дачный участок размером 20 м на 30 м. Определи площадь дачного участка, ответ запиши в квадратных сантиметрах.

Как найти площадь прямоугольника

Если вы считаете, что контент, доступный через Веб-сайт (как определено в наших Условиях обслуживания), нарушает
или другие ваши авторские права, сообщите нам, отправив письменное уведомление («Уведомление о нарушении»), содержащее
в
информацию, описанную ниже, назначенному ниже агенту. Если репетиторы университета предпримут действия в ответ на
ан
Уведомление о нарушении, оно предпримет добросовестную попытку связаться со стороной, которая предоставила такой контент
средствами самого последнего адреса электронной почты, если таковой имеется, предоставленного такой стороной Varsity Tutors.

Ваше Уведомление о нарушении прав может быть отправлено стороне, предоставившей доступ к контенту, или третьим лицам, таким как
в качестве
ChillingEffects.org.

Обратите внимание, что вы будете нести ответственность за ущерб (включая расходы и гонорары адвокатам), если вы существенно
искажать информацию о том, что продукт или действие нарушает ваши авторские права. Таким образом, если вы не уверены, что контент находится
на Веб-сайте или по ссылке с него нарушает ваши авторские права, вам следует сначала обратиться к юристу.

Чтобы отправить уведомление, выполните следующие действия:

Вы должны включить следующее:

Физическая или электронная подпись правообладателя или лица, уполномоченного действовать от их имени;
Идентификация авторских прав, которые, как утверждается, были нарушены;
Описание характера и точного местонахождения контента, который, по вашему мнению, нарушает ваши авторские права, в \
достаточно подробностей, чтобы позволить репетиторам университетских школ найти и точно идентифицировать этот контент; например нам требуется
а
ссылка на конкретный вопрос (а не только на название вопроса), который содержит содержание и описание
к какой конкретной части вопроса — изображению, ссылке, тексту и т. д. — относится ваша жалоба;
Ваше имя, адрес, номер телефона и адрес электронной почты; а также
Ваше заявление: (а) вы добросовестно полагаете, что использование контента, который, по вашему мнению, нарушает
ваши авторские права не разрешены законом, владельцем авторских прав или его агентом; (б) что все
информация, содержащаяся в вашем Уведомлении о нарушении, является точной, и (c) под страхом наказания за лжесвидетельство, что вы
либо владелец авторских прав, либо лицо, уполномоченное действовать от их имени.

Отправьте жалобу нашему уполномоченному агенту по адресу:

Чарльз Кон
Varsity Tutors LLC
101 S. Hanley Rd, Suite 300
St. Louis, MO 63105

Или заполните форму ниже:

Как найти длину и ширину прямоугольника с учетом площади

Обновлено 3 ноября 2020 г.

Крис Дезиел

Если вы знаете длину и ширину прямоугольника, вы можете определить его площадь.Однако эти две величины независимы, поэтому вы не можете произвести обратный расчет и определить их обе, если вам известна только площадь. Вы можете вычислить одно, если знаете другое, и вы можете найти их обоих в особом случае, когда они равны, что делает форму квадрата. Если вы также знаете периметр прямоугольника, вы можете использовать эту информацию, чтобы найти два возможных значения для длины и ширины.

Определение длины или ширины, когда вы знаете другое

Площадь прямоугольника ( A ) связана с длиной ( L ) и шириной ( W ) его сторон следующим соотношением:

A = L × W

Если вы знаете ширину, легко найти длину, переставив это уравнение, чтобы получить

L = \ frac {A} {W}

Если вы знаете длину и ширину, переставьте так, чтобы получить

W = \ frac {A} {L}

Пример: площадь прямоугольника составляет 20 квадратных метров, а его ширина — 3 метра.2} {3 \ text {m}} = 6.67 \ text {m}

Квадрат, особый случай

Поскольку у квадрата четыре стороны равной длины, площадь определяется как A = Л 2 . Если вы знаете площадь, вы можете сразу определить длину каждой стороны, потому что это квадратный корень из площади.

Пример: Какова длина сторон квадрата площадью 20 м 2 ?
Длина каждой стороны квадрата равна квадратному корню из 20, который равен 4.47 метров.

Определение длины и ширины, когда вы знаете площадь и периметр

Если вам известно расстояние вокруг прямоугольника, то есть его периметр, вы можете решить пару уравнений для L и W. Первое уравнение — это уравнение для площадь,

A = L × W

, а вторая — периметр,

P = 2L + 2W

Чтобы найти одну из переменных, скажем W , вы должны исключить другую. 2 — 8A}} {2}

Знание периметра может не дать вам однозначного ответа, но два ответа лучше, чем ничего.

Площадь прямоугольников — пояснения и примеры

По определению, площадь прямоугольника — это область, охватываемая прямоугольником в двухмерной плоскости . Прямоугольник — это двумерный многоугольник с четырьмя сторонами, четырьмя углами и четырьмя вершинами.

Прямоугольник состоит из двух сторон: длины (L) и ширины (W). Длина прямоугольника — самая длинная сторона, а ширина — самая короткая. Ширина прямоугольника иногда обозначается как ширина (b).

Как найти площадь прямоугольника?

Площадь прямоугольника можно рассчитать, подсчитав количество маленьких полных квадратов размером 1 * 1 кв. Единиц, необходимых для покрытия прямоугольника.

Например, если количество подсчитанных полных квадратов равно 20, это означает, что площадь прямоугольника равна 20 единицам квадратов.

Недостатком этого метода является то, что он не дает точных значений площади, а также метод неприменим для определения площади больших плоскостей.

Площадь прямоугольника Формула

Площадь прямоугольника равна произведению ширины и длины прямоугольника.

Следовательно, формула площади прямоугольника утверждает, что:

Площадь прямоугольника = Длина x Ширина

A = L * W, где A — площадь, L — длина, W — ширина или ширина. .

ПРИМЕЧАНИЕ: При умножении длины на ширину всегда проверяйте, что вы работаете в одной и той же единице длины. Если они даны в разных единицах, замените их на одну и ту же единицу.

Давайте решим несколько примеров задач о площади прямоугольника.

Пример 1

Найдите площадь прямоугольника, если его длина 25 м, а ширина 10 м.

Раствор

A = l x w

Заменить 25 на l и 10 на w.

= (25 x 10) м 2

= 250 м 2

Итак, площадь прямоугольника 250 м 2 .

Пример 2

Найдите площадь прямоугольника, длина и ширина которого составляют 10 см и 3 см соответственно.

Раствор

Дано,
Длина (l) = 10 см.
Ширина (б) = 3 см.
Площадь прямоугольника = длина × ширина

= 10 × 3 см 2 .

= 30 см 2 .

Пример 3

Если периметр прямоугольника составляет 60 см, а его длина в 5 раз больше ширины, найдите площадь прямоугольника.

Решение

Пусть ширина будет x.

Длина в 5 раз больше ширины, длина = 5x.

Но периметр прямоугольника = 2 (l + w) = 60 см

Замените 5x вместо l и x вместо w.

60 = 2 (5x + x)

60 = 12x

Разделите обе стороны на 12, чтобы получить.

x = 5

Теперь подставьте x = 5 в уравнение длины и ширины.

Следовательно, ширина = 5 см, а длина = 25 см.

Но площадь прямоугольника = lxw

= (25 x 5) см 2

= 125 см 2

Пример 4

Найдите площадь прямоугольника длиной 12 см и диагональ 13 см.

Решение

Здесь ширина не указана, поэтому мы используем теорему Пифагора для определения ширины.

c 2 = a 2 + b 2

13 2 = a 2 + 12 2

169 = a 2 + 144.

Вычесть 144 с обеих сторон .

169 — 144 = a 2 + 144 — 144

25 = a 2

Найдя квадратный корень из обеих сторон, мы получим.

a = 5

Следовательно, ширина прямоугольника 5 см.

Теперь посчитайте площадь.

A = Д x Ш

= (12 x 5) см 2

Пример 5

Если цена цементирования пола составляет 12,40 долларов США за квадратный метр, найдите стоимость цементирования прямоугольного пола длиной 20 м и шириной 10 м.

Раствор

Чтобы найти общую стоимость цементирования пола, умножьте площадь пола на коэффициент цементирования.

Площадь = Д x Ш

= (20 x 10) м 2

= 200 м 2

Стоимость цементирования = площадь x скорость цементирования

= 200 м 2 x 12,40 долл. США / м 2

= 2480 долларов США

Пример 6

Длина и ширина находятся в соотношении 11: 7, а его площадь составляет 693 квадратных фута. Найдите его длину и ширину.

Решение

Пусть общее соотношение длины и ширины = x

Следовательно, длина = 11x

Ширина = 7x

Площадь прямоугольника = L x W

693 кв.футов = (11x) (7x)

693 кв. ft = 77x 2

Разделим обе стороны на 77.

x 2 = 9

Найдите квадрат обеих сторон, чтобы получить;

x = 3.

Заменитель.

Длина = 11x = 11 * 3 = 33

Ширина = 7x = 7 * 3 = 21

Следовательно, длина прямоугольника составляет 33 фута, а его ширина — 21 фут.

Пример 7

Длина прямоугольника 0,7 м, ширина 50 см.Какова площадь прямоугольника в метрах?

Раствор

Длина = 0,7 м

Ширина = 50 см.

Преобразуйте 50 см в метры, разделив 50 на 100. Итак, 50 см = 0,5 м

Площадь = Д x Ш

= (0,7 x 0,5) м 2

= 0,35 м 2

Пример 8

Размер прямоугольной стены 75 м на 32 м. Узнайте стоимость покраски стены, если ставка покраски 5 рупий за кв.м.

Раствор

Площадь = Д x Ш

= (75 x 32) м 2

= 2400 м 2

Чтобы получить стоимость покраски стены, умножаем площадь стены по скорости покраски.

Стоимость = 2400 м 2 x 5 рупий за кв. М

= 12000 рупий

Пример 9

Прямоугольный пол двора, размером 50 м на 40 м, покрыт прямоугольной плиткой из размеры, 1 м на 2 м.Найдите общее количество плиток, необходимых для полного покрытия пола двора.

Решение

Сначала вычислите площадь пола двора и плитки.

Площадь пола двора = (50 x 40) м 2

= 2000 м 2

Площадь плитки = (1 x 2) м 2

= 2 м 2

Чтобы найти количество плиток, необходимое для покрытия пола двора, разделим пол двора на площадь плитки.

Количество плиток = 2000 м 2 /2 м 2

= 1000

Следовательно, для покрытия пола необходимо 1000 плиток.

Предыдущий урок | Главная страница | Следующий урок

Расчетная площадь | SkillsYouNeed

Площадь — это мера того, сколько места внутри фигуры. Расчет площади формы или поверхности может быть полезен в повседневной жизни — например, вам может потребоваться знать, сколько краски нужно купить, чтобы покрыть стену, или сколько семян травы вам нужно, чтобы засеять лужайку.

На этой странице описаны основные сведения, которые вам необходимо знать, чтобы понять и вычислить площади общих форм, включая квадраты и прямоугольники, треугольники и круги.

Расчет площади по сетке

Когда фигура рисуется на масштабированной сетке, вы можете найти площадь, подсчитав количество квадратов сетки внутри фигуры.

В этом примере внутри прямоугольника 10 квадратов сетки.


Чтобы найти значение площади с помощью метода сетки, нам нужно знать размер, который представляет квадрат сетки.

В этом примере используются сантиметры, но тот же метод применяется к любой единице длины или расстояния. Например, вы можете использовать дюймы, метры, мили, футы и т. Д.

В этом примере каждый квадрат сетки имеет ширину 1 см и высоту 1 см. Другими словами, каждый квадрат сетки равен одному квадратному сантиметру.

Подсчитайте квадраты сетки внутри большого квадрата, чтобы найти его площадь.

Есть 16 маленьких квадратов, поэтому площадь большого квадрата составляет 16 квадратных сантиметров.

В математике мы сокращаем «квадратные сантиметры» до 2 . 2 означает «в квадрате».

Каждый квадрат сетки равен 1 см 2 .

Площадь большого квадрата 16см 2 .


Подсчет квадратов на сетке для определения площади работает для всех форм — если известны размеры сетки. Однако этот метод становится более сложным, когда формы не точно соответствуют сетке или когда вам нужно подсчитать доли квадратов сетки.

В этом примере квадрат не точно помещается на сетке.

Мы все еще можем вычислить площадь, считая квадраты сетки.

  • Имеется 25 полных квадратов сетки (заштрихованы синим цветом).
  • 10 квадратов полусетки (заштрихованы желтым цветом) — 10 полуквадратов равны 5 полным квадратам.
  • Также есть 1 четверть квадрата (заштрихована зеленым) — (или 0,25 целого квадрата).
  • Сложите целые квадраты и дроби вместе: 25 + 5 + 0.25 = 30,25.

Следовательно, площадь этого квадрата составляет 30,25 см 2 .

Вы также можете записать это как 30¼см 2 .


Хотя использование сетки и подсчета квадратов внутри фигуры — очень простой способ изучения концепций площади, он менее полезен для нахождения точных областей с более сложными формами, когда можно сложить много частей квадратов сетки.

Площадь можно рассчитать с помощью простых формул, в зависимости от типа фигуры, с которой вы работаете.

Остальная часть этой страницы объясняет и дает примеры того, как вычислить площадь фигуры без использования системы сеток.


Площади простых четырехугольников:

квадратов, прямоугольников и параллелограммов

Простейшие (и наиболее часто используемые) вычисления площади выполняются для квадратов и прямоугольников.

Чтобы найти площадь прямоугольника, умножьте его высоту на ширину.

Для квадрата вам нужно только найти длину одной из сторон (поскольку каждая сторона имеет одинаковую длину), а затем умножить это на себя, чтобы найти площадь.Это то же самое, что сказать длину 2 или длину в квадрате.

Хорошей практикой является проверка того, что фигура на самом деле является квадратом, путем измерения двух сторон. Например, стена в комнате может выглядеть как квадрат, но когда вы ее измеряете, вы обнаруживаете, что на самом деле это прямоугольник.

Часто в реальной жизни формы могут быть более сложными. Например, представьте, что вы хотите найти площадь пола, чтобы заказать нужное количество ковра.

Типовой план помещения не может состоять из простого прямоугольника или квадрата:

В этом и других подобных примерах фокус состоит в том, чтобы разделить фигуру на несколько прямоугольников (или квадратов).Неважно, как вы разделите фигуру — любое из трех решений даст один и тот же ответ.

Для решений 1 и 2 необходимо создать две фигуры и сложить их площади, чтобы найти общую площадь.

Для решения 3 вы создаете большую форму (A) и вычитаете из нее меньшую форму (B), чтобы найти площадь.


Другая распространенная проблема — найти область границы — фигуры внутри другой фигуры.

В этом примере показана дорожка вокруг поля — ширина дорожки 2 метра.

Опять же, в этом примере есть несколько способов определить площадь пути.

Вы можете просмотреть путь как четыре отдельных прямоугольника, вычислить их размеры, а затем их площадь и, наконец, сложить области, чтобы получить итог.

Более быстрый способ — вычислить площадь всей формы и площадь внутреннего прямоугольника. Вычтите площадь внутреннего прямоугольника из всей, оставив площадь пути.

  • Площадь всей формы составляет 16 м × 10 м = 160 м 2 .
  • Мы можем определить размеры средней секции, потому что знаем, что путь по краю имеет ширину 2 метра.
  • Ширина всей формы составляет 16 м, а ширина пути по всей форме — 4 м (2 м слева от формы и 2 м справа). 16 м — 4 м = 12 м
  • То же самое для высоты: 10м — 2м — 2м = 6м
  • Итак, мы вычислили, что средний прямоугольник имеет размер 12 × 6 м.
  • Таким образом, площадь среднего прямоугольника составляет: 12 м × 6 м = 72 м 2 .
  • Наконец, мы убираем область среднего прямоугольника из области всей формы. 160 — 72 = 88м 2 .

Площадь тропы 88м 2 .


Параллелограмм — это четырехсторонняя форма с двумя парами сторон равной длины — по определению прямоугольник — это разновидность параллелограмма. Однако большинство людей склонны думать о параллелограммах как о четырехсторонних фигурах с наклонными линиями, как показано здесь.

Площадь параллелограмма рассчитывается так же, как и для прямоугольника (высота × ширина), но важно понимать, что высота означает не длину вертикальных (или отклоненных от вертикали) сторон, а расстояние между сторонами.

Из диаграммы вы можете видеть, что высота — это расстояние между верхней и нижней сторонами фигуры, а не длина стороны.

Представьте себе воображаемую линию под прямым углом между верхней и нижней сторонами. Это высота.


Области треугольников

Может быть полезно думать о треугольнике как о половине квадрата или параллелограмма.

Предполагая, что вы знаете (или можете измерить) размеры треугольника, вы можете быстро вычислить его площадь.

Площадь треугольника (высота × ширина) ÷ 2.

Другими словами, вы можете вычислить площадь треугольника так же, как площадь квадрата или параллелограмма, а затем просто разделите свой ответ на 2.

Высота треугольника измеряется по прямой линии от нижней линии (основания) до «вершины» (верхней точки) треугольника.

Вот несколько примеров:

Площадь трех треугольников на диаграмме выше одинакова.

Каждый треугольник имеет ширину и высоту 3 см.

Площадь рассчитана:

(высота × ширина) ÷ 2

3 × 3 = 9

9 ÷ 2 = 4,5

Площадь каждого треугольника составляет 4,5 см 2 .


В реальных ситуациях вы можете столкнуться с проблемой, которая требует от вас найти площадь треугольника, например:

Вы хотите покрасить фронтальный конец сарая. Вам нужно посетить магазин украшений только один раз, чтобы получить нужное количество краски.Вы знаете, что литр краски покроет 10 м 2 стены. Сколько краски нужно, чтобы покрыть фронтон?

Вам нужно три измерения:

A — Общая высота до вершины крыши.

B — Высота вертикальных стен.

C — Ширина здания.

В этом примере измерения:

A — 12,4 м

B — 6,6 м

C — 11,6 м

Следующий этап требует дополнительных расчетов.Подумайте о здании как о двух формах: прямоугольнике и треугольнике. По имеющимся у вас измерениям вы можете рассчитать дополнительное измерение, необходимое для определения площади фронтона.

Измерение D = 12,4 — 6,6

D = 5,8 м

Теперь вы можете определить площадь двух частей стены:

Площадь прямоугольной части стены: 6,6 × 11,6 = 76,56 м 2

Площадь треугольной части стены: (5.8 × 11,6) ÷ 2 = 33,64 м 2

Сложите эти две области вместе, чтобы получить общую площадь:

76,56 + 33,64 = 110,2 м 2

Как вы знаете, один литр краски покрывает 10 м 2 стены, поэтому мы можем рассчитать, сколько литров нам нужно купить:

110,2 ÷ 10 = 11,02 л.

На самом деле вы можете обнаружить, что краска продается только в 5-литровых или 1-литровых канистрах, результат — чуть более 11 литров. У вас может возникнуть соблазн округлить до 11 литров, но, если мы не будем разбавлять краску водой, этого будет недостаточно.Таким образом, вы, вероятно, округлите до следующего целого литра и купите две 5-литровые банки и две 1-литровые банки, что в сумме составит 12 литров краски. Это позволит избежать любых потерь и оставит большую часть литра для подкраски позднее. И не забывайте, что если вам нужно нанести более одного слоя краски, вы должны умножить количество краски для одного слоя на количество необходимых слоев!


Области кругов

Чтобы вычислить площадь круга, вам необходимо знать его диаметр или радиус .

Диаметр круга — это длина прямой линии от одной стороны круга до другой, проходящей через центральную точку круга. Диаметр в два раза больше длины радиуса (диаметр = радиус × 2)

Радиус круга — это длина прямой линии от центральной точки круга до его края. Радиус составляет половину диаметра. (радиус = диаметр ÷ 2)

Вы можете измерить диаметр или радиус в любой точке окружности — важно измерять, используя прямую линию, проходящую через (диаметр) или заканчивающуюся в (радиусе) центром окружности.

На практике при измерении окружностей часто легче измерить диаметр, а затем разделить на 2, чтобы найти радиус.

Радиус нужен для вычисления площади круга, формула:

площадь круга = πR 2 .

Это означает:

π = Pi — постоянная, равная 3,142.

R = радиус окружности.

R 2 (радиус в квадрате) означает радиус × радиус.


Следовательно, круг с радиусом 5 см имеет площадь:

3.142 × 5 × 5 = 78,55 см 2 .

Круг диаметром 3 м имеет площадь:

Сначала прорабатываем радиус (3м ÷ 2 = 1,5м)

Затем примените формулу:

πR 2

3,142 × 1,5 × 1,5 = 7,0695.

Площадь круга диаметром 3 м составляет 7,0695 м 2 .


Заключительный пример

В этом примере используется большая часть содержимого этой страницы для решения простых задач с областями.

Это дом Рубена М. Бенджамина в Блумингтоне, штат Иллинойс, внесенный в Национальный реестр исторических мест США (номер записи: 376599).

Этот пример включает поиск области фасада дома, деревянной решетчатой ​​части — исключая дверь и окна. Вам нужны следующие размеры:

A — 9,7 м B — 7,6 м
C — 8,8 м D — 4,5 м
E — 2.3 мес F — 2,7 м
G — 1,2 м H — 1,0 м

Примечания:

  • Все размеры являются приблизительными.
  • Не стоит беспокоиться о границе вокруг дома — она ​​не учтена в измерениях.
  • Мы предполагаем, что все прямоугольные окна одинакового размера.
  • Размер круглого окна — это диаметр окна.
  • Размер двери включает ступеньки.

Какова площадь деревянной реечной части дома?

Работы и ответы ниже:



Ответы на приведенный выше пример

Сначала определите площадь основной формы дома — прямоугольника и треугольника, составляющих форму.

Главный прямоугольник (B × C) 7,6 × 8,8 = 66,88 м 2 .

Высота треугольника (A — B) 9,7 — 7,6 = 2,1.

Следовательно, площадь треугольника равна (2.1 × C) ÷ 2.
2,1 × 8,8 = 18,48. 18,48 ÷ 2 = 9,24 м 2 .

Общая площадь фасада дома равна сумме площадей прямоугольника и треугольника:

66,88 + 9,24 = 76,12 м 2 .

Затем проработайте площади окон и дверей, чтобы их можно было вычесть из всей площади.

Площадь двери и ступенек составляет (Д × В) 4,5 × 2,3 = 10,35 м 2 .

Площадь одного прямоугольного окна составляет (G × F) 1.2 × 2,7 = 3,24 м 2 .

Есть пять прямоугольных окон. Умножьте площадь одного окна на 5.

3,24 × 5 = 16,2 м2. (общая площадь прямоугольных окон).

Круглое окно имеет диаметр 1 м, поэтому радиус 0,5 м.

Используя πR 2 , определите площадь круглого окна: 3,142 × 0,5 × 0,5 =. 0,7855м 2 .

Затем сложите площади двери и окон.

(зона двери) 10,35 + (прямоугольная зона окон) 16.2 + (площадь круглого окна) 0,7855 = 27,3355

Наконец, вычтите общую площадь окон и дверей из всей площади.

76,12 — 27,3355 = 48,7845

Площадь деревянного реечного фасада дома и ответ на проблему: 48,7845м 2 .

Вы можете округлить ответ до 48,8 м 2 или 49 м 2 .

См. Нашу страницу на Оценка, приближение и округление .

Калькулятор длины и ширины прямоугольника

Калькулятор длины и ширины прямоугольника — это бесплатный онлайн-инструмент, который помогает определить длину стороны прямоугольника.

Что такое длина и ширина калькулятора прямоугольника?

Прямоугольник — это замкнутая двумерная фигура с четырьмя сторонами и четырьмя углами. Длина противоположных сторон равна и параллельна друг другу. Калькулятор длины и ширины прямоугольника Cuemath — это бесплатный онлайн-инструмент, который рассчитывает длину и ширину прямоугольника за несколько секунд.

Примечание. Введите цифры до 3 цифр.

Как использовать калькулятор длины и ширины прямоугольника?

Выполните шаги, указанные ниже, чтобы найти длину и ширину прямоугольника.

  • Шаг 1: Введите значение площади и периметра прямоугольника в соответствующие поля ввода.
  • Шаг 2: Нажмите « Вычислить », чтобы найти соответствующую длину и ширину прямоугольника.
  • Шаг 3: Щелкните « Reset », чтобы очистить поля и ввести новые значения.

Как рассчитать длину и ширину прямоугольника?

Формула для площади «A» прямоугольника, длина и ширина которого равны «l» и «w» соответственно, является произведением длины и ширины, то есть « A = l × w ».

Формула для периметра P прямоугольника, длина и ширина которого равны l и w соответственно, равна P = 2 (l + w) .

Чтобы сначала вычислить длину и ширину прямоугольника, вычислите значение ширины ‘w’, используя формулу площади прямоугольника, то есть ‘ w = A / l’ . Затем подставьте значение ширины в формулу периметра прямоугольника и упростите значение длины l, то есть

.

P = 2 (l + A / I).

Затем подставьте значение длины «l» прямоугольника в уравнение w = A / l и выполните упрощение, чтобы найти ширину «w».

Давайте узнаем на примере, как найти длину и ширину прямоугольника.

Хотите найти сложные математические решения за секунды?

Воспользуйтесь нашим бесплатным онлайн-калькулятором для решения сложных вопросов. Cuemath находит решения простым и легким способом.

Забронируйте бесплатную пробную версию Класс

Решенный пример:

Если площадь и периметр прямоугольника составляют 8 квадратных единиц и 12 единиц соответственно.Какова длина и ширина прямоугольника?

Решение:

Подставьте значения площади в формулу «A = l × w» и упростите, чтобы найти ширину «w» в форме длины «l».

A = длина × ширина
8 = длина × ширина
ш = 8 / л

Подставьте значение ширины в формулу периметра прямоугольника и упростите значение длины l, то есть P = 2 (l + A / I).

P = 2 (д + ш)
12 = 2 (л + 8 / л)
6l = l 2 + 8
л 2 — 6 л + 8 = 0
l = 4 единицы (или) 2 единицы.

Подставьте значение длины «l» прямоугольника в уравнение w = A / l и упростите, чтобы найти ширину «w».

Вт = А / л
Если l = 4 единицы, то w = 8/4 = 2 единицы.
Если l = 2 единицы, то w = 8/2 = 4 единицы.

Следовательно, l = 4 единицы (или) 2 единицы
w = 2 единицы (или) 4 единицы
Теперь воспользуйтесь нашим онлайн-калькулятором длины и ширины прямоугольника и найдите длину и ширину прямоугольника, если его площадь и периметр следующие:

  • Площадь = 80 квадратных единиц, Периметр = 36 единиц
  • Площадь = 72 квадратных единицы, Периметр = 40 единиц

Калькулятор площади прямоугольника

Как рассчитать площадь прямоугольника

Если вам нужно найти площадь и периметр прямоугольника, этот калькулятор — удобный инструмент.

Просто введя длину и ширину, этот калькулятор почти мгновенно найдет периметр (P) и площадь (A).

Если вас интересуют калькуляторы для множества других фигур, вы можете взглянуть на другие наши удобные калькуляторы. Но вы можете остаться здесь и узнать больше о том, как найти площадь прямоугольника.

Прямоугольник имеет четыре угла по 90 градусов. Если длины сторон одинаковы, то прямоугольник также является квадратом. Длины сторон будут указаны как a или b , или вы можете использовать l и w для «длины» и «ширины».Диагональ, которая идет от одной вершины к противоположной вершине, разделяющей прямоугольник на два квадрата, называется диагональю и обозначается как d .

Вот основные формулы, используемые калькулятором.

Площадь (A) = a (b)

Периметр (расстояние по внешней стороне прямоугольника) = a + a + b + b или 2 a + 2 b и обозначается как (P)

Диагональ составляет d ² = a ² + b ², что является теоремой Пифагора (см. наш калькулятор теоремы Пифагора).

Пример вычисления площади прямоугольника:

Предположим, что длина a = 6 дюймов и ширина b = 4 дюйма

A = a * b , поэтому A = 6 (4) = 24 дюйма²

Используя те же размеры, мы можем рассчитать периметр.

Периметр равен 2 a + 2 b , поэтому в этом примере периметр

P = 2 (6) + 2 (4) = 20 дюймов

Чтобы найти диагональ, используя те же размеры:

d ² = 6² + 4² = 36 + 16 = 52

Извлеките квадратный корень из обеих сторон и диагональ d будет приблизительно равна 7.2 дюйма

Эти примеры иллюстрируют, как вычислить площадь, периметр и диагональ прямоугольника вручную, но если вы предпочитаете использовать калькулятор для более быстрых результатов или просто для проверки своей работы, то не стесняйтесь делать это. Отличной особенностью калькулятора является то, что вы можете определить длину или ширину, если знаете периметр и длину одной из сторон.

Площадь, периметр и диагональ прямоугольника

На этой странице показано, как измерить площадь прямоугольника.Мы рассмотрим следующие темы:

Какова площадь, периметр и диагональ прямоугольника?

Как рассчитать площадь, периметр и диагональ прямоугольника?

Реальное приложение для вычисления площади, периметра и диагонали прямоугольника

Площадь прямоугольника

Представьте себе площадь прямоугольника в виде квадратов внутри прямоугольника. Прямоугольник ниже имеет покрытую площадь 12 «квадратных единиц»

Пространство внутри двухмерной формы — это площадь или количество закрытых форм.

На этой диаграмме показаны ширина, длина и площадь прямоугольника:


Расчет площади прямоугольника

Чтобы найти площадь прямоугольника, вам нужно умножить длину и ширину прямоугольника. . Мы можем получить площадь прямоугольника по следующей формуле:

A = L * W

A — площадь, L — длина, а W — ширина.

Пример 1

Вычислите площадь прямоугольника длиной 7 сантиметров и шириной 5 сантиметров.

Формула:

A = L * W

Ответ:

A = 35. Заданная длина ( L ) равна 7, а 3 — ширина ( W ). При умножении вы получите 35 как свою площадь.

Периметр прямоугольника

Посмотрите на изображение ниже, человек ходит вокруг коробки. Путь, по которому он ходит от начальной точки и обратно, — это периметр.Зная длину и ширину прямоугольника, теперь мы можем получить периметр прямоугольника. Обе противоположные стороны прямоугольника совпадают, что означает, что, сложив эти стороны, мы можем рассчитать периметр.


Расчет периметра прямоугольника

Теперь, сложив все стороны прямоугольника, мы можем получить периметр. Вот уравнение для получения периметра прямоугольника:

P = L + W + L + W

Поскольку мы знаем, что обе противоположные стороны прямоугольника идентичны, мы можем упростить уравнение, используя это уравнение :

P = 2L + 2W

Пример 1

Найдите периметр прямоугольника длиной 12 см и шириной 7 см.

Формула:

P = L + W + L + W или

P = 2L + 2W

Ответ:

P = 12 + 7 + 12 + 7 или

P = 2 (12) + 2 (7)

Ответ: P = 38. Добавляя 12 ( L ) + 7 ( W ) + 12 ( L ) + 7 ( W ) , вы получите 38. Умножив длину ( L ) и ширину ( W ) на 2, а затем сложив частные, вы получите тот же ответ.

Диагональ прямоугольника

Если присмотреться, прямоугольник представляет собой комбинацию двух прямых углов. Диагональ — это разделение прямоугольника на два прямоугольных треугольника, идентичных друг другу.


Расчет диагонали прямоугольника

Мы знаем, что прямоугольник — это комбинация двух прямоугольных треугольников. Диагональ этого прямоугольника — это гипотенуза двух треугольников, , поэтому мы можем применить теорему Пифагора для определения диагонали прямоугольника.{2}} \)
\ (D = \ sqrt {34} \)

D = 5,83

Реальные приложения для определения площади прямоугольника

Молодожены хотят укладывать плитку на пол дома. главная спальня. Комната имеет длину 20 футов и ширину 30 футов. Плитка, которую они выбрали, имеет длину 24 дюйма и ширину 36 дюймов. Определите количество плиток, необходимых для заполнения главной спальни.

Советы:

1. Определите площадь, занимаемую главной спальней.

2. Вычислите площадь плитки

3. Выберите единицу измерения, которая будет использоваться. В этом примере будут использоваться ножки.

Решение:

A = Д * Ш

A = 20 футов x 30 футов

A = 600 футов²

Наконечник:

Прежде чем мы получим площадь каждой плитки, преобразуйте футы в дюймы

Пример преобразования:

1 фут = 12 дюймов

2 фута = 24 дюйма

3 фута = 36 дюймов

Площадь 1 плитки = Д × Ш

Площадь 1 плитки = 2 × 3

Площадь 1 плитки = 6 футов²

Это означает, что каждая плитка имеет площадь 6 футов², и он должен покрывать площадь комнаты, составляющей 600 футов².Итак, 6 × 100 = 600.

Количество плиток, необходимых для заполнения главной спальни, составляет 600.

Реальные приложения для определения периметра прямоугольника

Фермер хочет добавить клетку для цыплят. Он хочет добавить новый забор возле своего дома и свободное пространство длиной 30 метров и шириной 16 метров. Найдите периметр свободного места.

Решение:

P = 2L + 2W

Ответ:

Периметр = 2 (длина свободного пространства) + 2 (ширина свободного пространства)

P = 2 (30) + 2 (16)

P = 60 + 32

P = 92

Периметр свободной площади 92 метра.{2}} \)

\ (D = \ sqrt {236} \)

D = 14,42

Чтобы равномерно разделить сэндвич на два равных прямоугольных треугольника, длина диагонали должна быть 14,42 дюйма.

Обнаружение формул площади — элементарная математика

Формулы площади

Учащиеся, неформально относящиеся к области как к «количеству двумерного« материала », содержащегося внутри области, могут придумать для себя большинство формул, которые их часто просят просто запомнить.Каждая формула, которую они изобретают заново, помогает им лучше понять (и запомнить) другие формулы, которые они знают. (См. Также площадь поверхности.)

Площадь прямоугольников

Выбирая квадрат в качестве единицы площади, мы получаем интуитивное представление о площади прямоугольников. Если мы решим, что площадь этого квадрата равна 1, то прямоугольник, который в 7 раз длиннее, будет иметь площадь 7 × 1.

Прямоугольник, который в два раза больше высоты, будет иметь удвоенную площадь, поэтому площадь составляет 2 × 7 единиц площади.Мы можем сосчитать два ряда по семь квадратов. Точно так же он имеет 3 ряда по 7 квадратов (или 7 столбцов по 3 квадрата), всего 7 × 3 квадратов, поэтому его площадь составляет 21 квадратную единицу.

Количество квадратов в одном ряду — это длина прямоугольника. Количество строк — это высота прямоугольника. Таким образом, площадь равна длине × высоте.

Поскольку прямоугольник можно нарисовать под наклоном, «высота» определяется как «направление, перпендикулярное основанию», а «основание» определяется как, ну, с какой бы стороны вы ни выбрали его.

Это работает для подсчета чисел. Это работает даже для дробей. Показанный здесь синий прямоугольник измеряет половину единицы длины на пять с половиной единиц длины. Если мы выберем соответствующий квадрат в качестве единицы площади, мы увидим, что синий прямоугольник содержит пять половинных единиц площади и одну четверть единицы площади, или всего две и три четверти единицы площади. (Розовые части показывают завершение каждой квадратной единицы площади.)

Чтобы включить всех чисел, мы определяем площадь прямоугольника как основание × высота (где «основание» и «высота» означают длин этих сторон, измеряется в те же единицы ).

Площадь параллелограммов

Идея

Мы можем вычислить формулу площади параллелограмма, разрезав параллелограмм и переставив части, чтобы получился прямоугольник. Поскольку параллелограмм и прямоугольник состоят из одинаковых частей, они обязательно имеют одинаковую площадь. (См. Определение площади, чтобы узнать, почему эти области одинаковы.)

Мы видим, что и имеют точно такую ​​же длину основания (синий) и точно такую ​​же высоту (зеленый).Поскольку основание × высота дает площадь прямоугольника, мы можем использовать те же измерения на параллелограмме, чтобы вычислить его площадь: основание × высота . (Как и раньше, «высота» измеряется перпендикулярно основанию, а «основание» — это та сторона, которую вы выбрали первой. См. Параллелограмм.)

На разрезе, показанном выше, легко увидеть, что базовая длина не изменилась.Фактически, перпендикулярный разрез можно делать где угодно по основанию.

Укрепление отверстий

Интуиция и доказательство

Это рассечение дает интуитивное понимание формулы площади параллелограмма, причина , что он должен быть тем, чем он является. Но мы не сомневались, действительно ли рассечение «работает». То есть, когда мы разрезаем параллелограмм и переставляем его части, мы ожидаем, что получит , и результат определенно будет таким.Но внешность может быть обманчива. Что убеждает нас в том, что когда мы перемещаем этот треугольник, в результате получается прямоугольник? Что, если это больше похоже (хотя и менее преувеличено)? Если результат не всегда является идеальным прямоугольником, мы не можем использовать наши знания формулы площади прямоугольника, чтобы разработать формулу для параллелограмма. В старших классах учащиеся смогут доказать, что две части параллелограмма, если правильно собрать их, действительно образуют прямоугольник. В классах K-8 учащиеся по большей части должны полагаться на визуальный эксперимент и получать интуитивное ощущение.Узнайте больше о том, почему эти вскрытия работают.

Что, если мы выберем короткую сторону в качестве основания?

Мы можем выбрать любую сторону в качестве базы; «Высота» определяется как измеряемая перпендикулярно стороне, которую мы выбираем в качестве основания. Если мы возьмем за основу короткую сторону (синюю), то рассечение, показанное выше, не будет таким убедительным. Резка на такой высоте и перестановка деталей оставляет беспорядок:

В этом конкретном примере мы можем спасти беспорядок, сделав еще один разрез, но что, если бы параллелограмм был еще длиннее и тоньше?

Оказывается, что любой параллелограмм , независимо от его длины и тонкости, можно разрезать таким образом, чтобы части — возможно, многие из них — могли быть преобразованы в прямоугольник.Но нужно потрудиться, чтобы показать, что это всегда можно сделать. Нам нужна другая идея.

Немного другая идея вскрытия в этом случае значительно упрощает жизнь. (Самостоятельно вы можете показать, что это работает и в исходном случае.)

  • Заключите параллелограмм в прямоугольник.
  • Две части прямоугольника, которые составляют , а не внутри параллелограмма, являются конгруэнтными треугольниками.
  • Сдвиньте один из этих треугольников по направлению к другому, пока они не встретятся, образуя прямоугольник.Поскольку общая площадь внешнего прямоугольника не изменилась (это тот же прямоугольник, что и раньше), а желтая область не изменилась (части просто перемещались), разница между ними — фиолетовыми областями — должна быть такой же. Как и раньше, мы также можем видеть, что размеры прямоугольной фиолетовой области являются основанием и высотой исходного параллелограмма.

Интуиция и доказательство, повторение: Опять же, рассечение дает существенное понимание, но требуется немного больше работы, чтобы убедиться, что два желтых треугольника, которые, безусловно, выглядят , как будто они соединяются вместе, образуя прямоугольник, на самом деле точно подходят, а не только почти .

Почему так важно быть осторожным?

При построении других формул площади (см. Ниже) мы захотим, чтобы использовал как наш способ определения площади параллелограмма, и поэтому мы хотим иметь возможность полагаться на найденное нами правило. Мы, , можем быть уверены, что перестановка частей не приведет к изменению области: это, в конце концов, то, как мы определяем область . Но мы также должны быть уверены, что детали подходят друг к другу, как мы заявляем, что это , иначе мы не можем полагаться на сделанные нами измерения.И мы должны быть уверены, что правило base × height не зависит от удачного выбора базы.

В большинстве учебных программ учащиеся не имеют достаточной систематической базы геометрических знаний до 8-го класса, чтобы убедительно доказать, что эти вскрытия работают. Но интуитивного понимания достаточно для объяснения и обоснования формул и хорошей основы для дальнейшего изучения геометрии.

Площадь треугольника

Знание того, как найти площадь параллелограмма, помогает нам найти площадь треугольника.

Рассечение треугольника

Мы можем разрезать треугольник на две части — одну из них треугольник, а другую — трапецию, разрезав его параллельно основанию. Если мы разрежем высоту ровно пополам с этим срезом, две части соединятся, образуя параллелограмм с таким же основанием , но половиной высоты .

Итак, основание × на полувысоте дает площадь треугольника. Подобный разрез показывает полубазовая × высота .Любой из них сокращается до bh .

Удвоение треугольника и уменьшение полученной площади вдвое

Другой способ мышления: две копии треугольника образуют параллелограмм с тем же основанием, и такой же высотой , что и треугольник.

Площадь параллелограмма равна основанию × высоте , но это вдвое больше площади треугольника, поэтому площадь треугольника составляет из основания × высоты , как мы видели с методом рассечения.

(Как всегда, выберите «основание» и измерьте высоту перпендикулярно этому основанию, от основания до противоположной вершины.)

Площадь трапеции

Удвоение трапеции и уменьшение полученной площади вдвое

Как и в случае с треугольником, две копии трапеции можно соединить вместе, чтобы получился параллелограмм.

Высота параллелограмма равна высоте трапеции, но его основание равно сумме двух оснований трапеции.Таким образом, площадь параллелограмма равна по высоте × ( base1 + base2 ). Но эта площадь составляет две трапеции, поэтому нам нужно разрезать ее пополам, чтобы получить площадь трапеции.

Рассечение трапеции

Мы также можем разрезать трапецию так же, как мы разрезали треугольник, с одним срезом, разрезающим ее высоту пополам. Две части соединяются вместе, образуя параллелограмм, основание которого является суммой двух оснований трапеции, но высота которого равна половине высоты трапеции.

В случае трапеции основания не могут быть выбраны произвольно. Две параллельные стороны являются основаниями, а высота, как всегда, представляет собой перпендикулярное расстояние от одного основания до противоположного.

Площадь этого параллелограмма равна его высоте (полувысоте трапеции), умноженной на его основание (сумма оснований трапеции), поэтому его площадь составляет полувысоты × ( base1 + base2 ). Поскольку параллелограмм состоит из того же материала, что и трапеция, это тоже площадь трапеции.

В любом случае площадь трапеции составляет × по высоте × ( base1 + base2 ) .

Площадь других специальных четырехугольников

Площадь ромба

Площадь ромба можно найти, разрезав и переставив части, чтобы они образовали параллелограмм. Это можно сделать несколькими способами:

  1. Разрежьте более короткую диагональ (a), чтобы получились два равных треугольника. Переместите нижнюю половину треугольника рядом с верхней половиной, чтобы получился параллелограмм.Более короткая диагональ (a) становится основанием параллелограмма, а половина более длинной диагонали (b) становится высотой параллелограмма. Таким образом, площадь ромба равна a * b или произведению диагоналей, что является стандартной формулой для ромба.
  2. Другой аналогичный способ — разрезать ромб на четыре равных треугольника и переставить их в прямоугольник с более короткой диагональю в качестве основания и половиной длинной диагонали в качестве высоты.
  3. После разрезания ромба на два равных треугольника мы можем вычислить площадь одного из треугольников, которая равна * основанию (a) * высоте (b) = ab.Затем умножьте на два, так как их два: 2 * ab = ab.

Площадь кайта

Площадь воздушного змея может быть похожа на площадь ромба. Если разрезать более длинную диагональ, получатся два равных треугольника. Если мы переставим их, мы можем сформировать параллелограмм с более длинной диагональю (b) в качестве основания и половиной более короткой диагонали (a) в качестве высоты. Итак, площадь становится b * a = ab. Более сложный подход требует немного алгебры. Разрежьте змей по более короткой диагонали, чтобы сформировать два треугольника с более короткой диагональю (a) в качестве основы.Таким образом, площадь первого треугольника — * волнистая, где волнистая — высота. Площадь второго треугольника равна a * (b — волнистый), где (b — волнистый) — оставшаяся часть большей диагонали. Таким образом, общая площадь становится (a * волнистая) + (a * (b — волнистая)). Вынося за скобки a, получаем a (волнистый + b — волнистый) = ab.

Ну что вы знаете. По сути, вам нужно только знать формулу для площади параллелограмма, а затем вывести формулу для других.

.

Добавить комментарий

Ваш адрес email не будет опубликован. Обязательные поля помечены *